PANCE exam c part 2

Ace your homework & exams now with Quizwiz!

A 3-year-old boy presents after swallowing a quarter. He now feels it is stuck. At which location is an obstruction most likely to occur? AAortic arch BCricopharyngeus muscle CGastroesophageal junction DTracheal bifurcation

Cricopharyngeus muscle based on physiological narrowing order

A 43-year-old-man with known alcoholic cirrhosis presents to the emergency department with fever and abdominal pain. His family notes he has also been more confused and sleepy than usual. He has not been feeling well for a few days. He denies trauma, change bowel habits, and blood in his stool. His abdomen is distended with a fluid wave and is diffusely tender to palpation. What is the most likely diagnosis? AAppendicitis BCholecystitis CColitis DSpontaneous bacterial peritonitis

Spontaneous bacterial peritonitis Diagnosis of spontaneous bacterial peritonitis is based on analysis of the ascitic fluid. A total WBC count > 1000 cells/mm3 or a neutrophil count > 250 cells/mm3 in ascites fluid is diagnostic for spontaneous bacterial peritonitis. Antibiotic treatment with a third generation cephalosporin is the first line and albumin may reduce progression of hepatorenal syndrome. Spontaneous Bacterial Peritonitis History of chronic liver disease or cirrhosis Fever, chills, and abdominal pain PE will show ascites, shifting dullness Labs will show PMNs > 250/µL, WBC > 1,000/µL, pH < 7.34 Diagnosis is made by analysis of the ascitic fluidIf due to peritoneal dialysis - two of the following abdominal pain or cloudy peritoneal fluid ≥ 100 WBCs/mm3 with 50% PMNs positive culture Most commonly caused by gram-negative rods (eg, E.coli, Klebsiella), Streptococcus Treatment is immediate IV antibiotics (third-generation cephalosporin), consider giving albuminIf due to peritoneal dialysisvancomycin for gram-positive and MRSA coverage with a third- or fourth-generation cephalosporin(ie, cefepime), intraperitoneal administration preferred

A 2-month-old boy presents with a low fever and cough. Which of the following is suggestive of Chlamydia pneumonia in this infant? ABullous myringitis BDiarrhea CRusty-colored sputum DStaccato cough

Staccato cough Patient will be between 3 and 19 weeks old History of congestion Cough, hoarseness, and malaise PE will show staccato cough and a lack of fever Most commonly caused by Chlamydia trachomatis Treatment is macrolide antibiotic

Which of the following characteristics suggests transfusion-related acute lung injury instead of transfusion-associated circulatory overload?👀👀👀 AHigh fever BHypertension CHypoxemia DPulmonary edema

High fever Fever is not common in transfusion-associated circulatory overload (TACO). Transfusion Complications Massive transfusion: coagulopathy, hypothermia, hypocalcemia Febrile reaction: most common complication, fever or chills Hemolytic reaction: ABO incompatibility, immediate fever or chills, HA Rx: stop transfusion, IVF, diuretics Allergic reaction: urticaria or hives TRALI: like ARDS Rx: stop transfusion TRALI is characterized by hypoxemia (oxygen saturation < 90% on room air), and bilateral pulmonary infiltrates presenting within 6 hours of blood transfusion. There should not be any evidence of pulmonary artery hypertension or generalized fluid overload. Hypotension, tachycardia, and fever are common. TACO: pulmonary edema due to circulatory overloadRx: stop transfusion, diuresis Delayed reaction: 3-4 weeks after transfusion, decreased Hgb GVHD: immunocompromise, rash, pancytopenia, increased LFTs Prevention: irradiated blood products in immunocompromised

A 44-year-old man with a history of hypertension presents with purple, polygonal, papular lesions on the wrists. The lesions are pruritic and have been present for the last several weeks. A skin biopsy is ordered. Which of the following histology results would be most consistent with the suspected diagnosis? AHomogenized superficial dermis with a flattened dermoepidermal junction BHyperkeratosis, degeneration of the basal cell layer and colloid bodies CHyperkeratosis, parakeratosis, and acanthotic epidermis DLeukocytoclasia, nuclear dust, and endothelial cell swelling

Hyperkeratosis, degeneration of the basal cell layer and colloid bodies dx: lichen planus -------------------------------------------------------------------- vs Homogenized superficial dermis with a flattened dermoepidermal junction (A) is histologically characteristic of lichen sclerosus et atrophicus and is not typical for lichen planus. Hyperkeratosis, parakeratosis, and acanthosis (C)is characteristic of psoriasis. Leukocytoclasia, nuclear dust, and endothelial cell swelling (D) accompanies red cell extravasation seen in vasculitis.

A 33-year-old man presents to the emergency department with a left eye injury that occurred just prior to arrival. Patient states that he was working with a table saw and a piece of wood flew into his left eye. He has decreased visual acuity, eye pain, profuse tearing, photophobia, and difficulty opening his eye. He has a foreign body sensation as well. Which of the following findings on physical exam would suggest globe perforation? AA negative Seidel test BConjunctival injection CHyphema in the anterior chamber DIncrease intraocular eye pressure

Hyphema in the anterior chamber

What is the most frequently isolated organism in acute suppurative parotitis? ABacteroides sp. BEscherichia coli CStaphylococcus aureus DViridans streptococci

Staphylococcus aureus

A 24-year-old man presents to the emergency department via ambulance after having two witnessed seizures at home. The patient received two doses of intravenous lorazepam en route but is still exhibiting seizure-like activity. You decide to administer 1,500 mg of intravenous phenytoin. Over how many minutes should the medication be infused? A10 minutes B30 minutes C5 minutes D60 minutes

30 minutes A general rule to follow when administering phenytoin intravenously is to infuse no faster than 50 mg/min. Since this patient is receiving 1,500 mg, this means that the fastest infusion time should be 30 minutes. ---------------------------------------------------------------- vs 10 minutes (A) and 5 minutes (C) are both incorrect as these infusion times are too fast and could lead to the aforementioned bradycardia and hypotension. 10 minutes would be correct if fosphenytoin was being administered, as this is a water-soluble form of phenytoin that can be infused at a maximum rate of 150 mg/min. Phenytoin can be administered safely over 60 minutes (D) for this particular dose but is unnecessary and delays treatment in a patient who is still actively seizing.

Which of the following patients meet criteria for metabolic syndrome? AA 35-year-old man with a waist circumference of 108 cm, triglycerides of 130 mg/dL, and blood pressure of 140/90 mm Hg BA 40-year-old woman with a body mass index of 35, triglycerides 150 mg/dL, and fasting glucose of 110 mg/dL CA 45-year-old man with a waist circumference of 110 cm, triglycerides of 155 mg/dL, and fasting glucose of 120 mg/dL DA 50-year-old man with waist circumference of 110 cm, triglycerides of 150 mg/dL, and high density lipoprotein cholesterol of 50 mg/dL

45-year-old man with a waist circumference of 110 cm, triglycerides of 155 mg/dL, and fasting glucose of 120 mg/dL metabolic syndrome is defined by any three of the following criteria: (1) Waist circumference of > 102 cm for men and > 88 cm for women, (2) Triglycerides of > 150 mg/dL, high density lipoprotein (HDL) cholesterol of < 40 for men and < 50 mg/dL for women, blood pressure of > 130/ > 85 mm Hg, and (3) Fasting glucose of > 100 mg/dL. Therefore, the 45-year-old man with a waist circumference of 110 cm, triglycerides of 155 mg/dL, and fasting glucose of 120 mg/dL meets criteria for metabolic syndrome.

A 3-year-old boy presents with severe vomiting and diarrhea. The exam reveals sunken eyes, skin tenting, and a capillary refill of three seconds. At his last well visit he weighed 15 kg, but weighs just 13 kg today. Using the Holliday-Segar method, what is his maintenance fluid rate per hour? A260.0 mL/hr B39.0 mL/hr C47.9 mL/hr D83.3 mL/hr

47.9 mL/hr This method is sometimes shortened to the 4-2-1 method of 4 mL/kg/hr for the first 10 kg, 2 mL/kg/hr for the next 10 kg, and 1 mL/kg/hr for each additional kilogram. (4 mL/hr x 10 kg) + (2 mL/hr x 3 kg) = 46 mL/hr or Holliday-Segar method of 100 mL/kg/day for the first 10 kg, 50 mL/kg/day for the next 10 kg, and 20 mL/kg/day for each additional kilogram. The total is then divided by 24 hours. (100 mL/kg/day x 10 kg) + (50 mL/kg/day x 3kg) = 1150 mL/day/24hr = 47.9mL/hr

Nonweight-bearing and pain control dxpelvic avulsion fracture of the anterior inferior iliac spine (AIIS) Pelvic Avulsion Fracture Powerful contraction of lower leg muscle on a developing apophysis Most common type: ischial tuberosity avulsion Diagnosed with AP and frog-leg radiographs Treated with rest, crutch use, and pain control Surgical pinning rarely required

A 12-year-old girl presents with right hip pain after kicking a ball to score at her soccer game. She describes hearing a pop followed by immediate pain. The patient can limp and localizes pain to the anterior right hip. An X-ray is obtained, as seen above. Which of the following is the most appropriate management? AImmobilization with a spica cast BMuscle relaxant and heat CNonweight-bearing and pain control DSurgical pinning

Discharge home with repeat beta-hCG in 48 hours dx:ectopic pregnancy. ---------------------------------------------------------------------------- Rhogam (B) and (C) is recommended for patients who are Rh-negative and have vaginal bleeding. If the mother is exposed to fetal blood, she may develop antibodies that threaten future pregnancies. This patient does not have vaginal bleeding and is Rh-positive obviating the need for Rhogam. Methotrexate (A) is a chemotherapeutic agent that can be used for the treatment of early ectopic pregnancy. This approach is not indicated in a wanted pregnancy with a beta-hCG below the discriminatory zone as repeat testing may show a viable intrauterine pregnancy.

A 26-year-old woman presents with abdominal cramping after a positive home pregnancy test. She states that the pregnancy is wanted. Her vital signs are T 98.7°F, HR 94 bpm, BP 110/66 mm Hg, RR 18/min, and SpO2 97%. Her exam is unremarkable. Labs reveal a serum beta-hCG of 1,000 mIU, and she is Rh-positive. An ultrasound is performed, as seen above. Which of the following is the appropriate management for this patient? AAdminister methotrexate BAdminister Rhogam and discharge home with repeat beta-hCG in 48 hours CAdminister Rhogam and methotrexate DDischarge home with repeat beta-hCG in 48 hours

Analgesia and a clean dressing dx herpetic whitlow Management • Self-limited • Do NOT perform incision and drainage • Acyclovir

A 27-year-old man presents with painful, swelling of his fingertip for three days as shown above. He also reports subjective fever and malaise. Which of the following represents the best course of management? AAnalgesia and a clean dressing BBedside incision and drainage CIntravenous antibiotic administration and hospital admission DOperative open irrigation and debridement

Recent episode of otitis media #ring enhanced lesion dx: Brain abscesses are infections of the brain parenchyma that can occur via direct spread or hematogenous seeding. Brain abscesses are often a complication from another infection, such as a recent episode of otitis media, sinusitis, or a dental infection. The symptoms of brain abscess include fever, headache, and focal neurological deficits. The headache is often severe and localized to the site of the abscess.

A 3-year-old boy presents to the emergency department via EMS after experiencing a seizure at home. His parents explain that the child had been having headaches for the past 3 days as well as fatigue. The child seized for approximately 4 minutes and since then has been fatigued and drowsy. This is the first time the child has experienced a seizure. Upon arrival at the emergency department, the patient's vital signs are a BP of 100/60 mm Hg, HR of 100 bpm, RR of 20/min, oxygen saturation of 96%, a T of 102.8°F, and a Glasgow Coma Scale score of 14. Upon physical examination, the patient is drowsy but is oriented to person, place, time, and event. Mild unilateral facial muscle weakness is noted when the patient tries to smile. Kernig and Brudzinksi tests are performed and are negative. The rest of the physical examination is normal. A head CT is obtained and is shown above. Which historical detail is most relevant in establishing the diagnosis? AFamily history of seizure BHistory of gastroenteritis CMaternal history of perinatal infection DPresence of lead paint in the home ERecent episode of otitis media

Spontaneous hemorrhage dx: Arteriovenous malformations (AVMs) of the brain are rare occurrences but can cause significant morbidity and mortality. ------------------------------------------------------------------------- vs Blindness (A) may occur in a patient with a hemorrhaging AVM of the occipital lobe or via mass effect if the lesion is near the optic chiasm or the optic nerve. The lesion in the above MRI is not near these areas. Flaccid paralysis (B) occurs in lower motor neuron disorders. In general, upper motor neuron disease and cerebral disease cause spastic paralysis due to the loss of central inhibition of lower motor neuron firing. Growth into several other cranial regions (C) is rarely seen in a patient with an AVM. Some patients have multiple malformations, especially patients with hereditary hemorrhagic telangiectasia, but these lesions would be detected at the same time as the presenting lesion. Spontaneous resolution (E) of AVMs can occur but is rare.

A 33-year-old man presents to the clinic reporting chronic headaches. The headaches are preceded by a visual aura and cause nausea, vomiting, and photosensitivity. His medical history is positive only for a tonsillectomy. The patient takes no medications except ibuprofen as needed for headaches. His vital signs are within normal limits. CMP, CBC, and TSH are all within normal limits. An MRI of the head is shown above. Which of the following is likely to occur in the future if this patient's findings on head imaging are left untreated? ABlindness BFlaccid paralysis CGrowth into several other cranial regions DSpontaneous hemorrhage ESpontaneous resolution

Insert a nasogastric tube Bowel adhesions following abdominal surgery are the most common cause of small bowel obstruction. Management begins with restricting oral intake and inserting a nasogastric tube to decompress the stomach and bowel. Sometimes this will relieve the obstruction if it is partial (gas seen in the colon). However, complete small bowel obstruction usually requires surgical correction. Administering intravenous fluids and correcting any electrolyte imbalance should also occur immediately.

A 43-year-old man is admitted to the hospital with nausea, vomiting, and intermittent diarrhea for the past two weeks. He has a history of abdominal surgery following a motor vehicle crash five years ago. Vital signs are BP 120/70, HR 80, RR 14, and T 37.1°C. On physical exam, you note a distended abdomen. A plain film abdominal X-ray is seen above. Which of the following is the most appropriate step in management? AAdminister antibiotics BInsert a nasogastric tube CPerform a paracentesis DPerform manual fecal disimpaction

Columnar-lined surface epithelium with goblet cells Barrett Esophagus History of chronic reflux Diagnosis is made by upper endoscopy, biopsy Biopsy will show squamous to columnar epithelium and proximal shift in the squamocolumnar junction Treatment is proton pump inhibitors Increased risk for adenocarcinoma

A 54-year-old man with a history of gastroesophageal reflux disease and hypertension presents to the office for a new patient physical exam. He currently takes omeprazole 20 mg twice daily and lisinopril 10 mg once daily. He has taken omeprazole for 5 years but states he has struggled with heartburn for almost 15 years. He also reports a 20 pack-year smoking history. His vital signs are heart rate 70 bpm, blood pressure 125/85 mm Hg, respirations 20/min, oxygen saturation 98% on room air. His physical exam is normal, with no abdominal tenderness to palpation. You order an upper endoscopy, which shows the findings represented in the image above. According to the American College of Gastroenterology, what endoscopy findings are consistent with the most likely diagnosis? AColumnar-lined surface epithelium with goblet cells BGastric cardiac-type epithelium CIntestinal metaplasia at the gastroesophageal junction DMetaplastic esophageal columnar mucosa EStratified squamous epithelium

Synchronized cardioversion at 200 joules dx; unstable vtach Ventricular Tachycardia More than three consecutive ectopic ventricular beats Monomorphic, polymorphic Bidirectional: digoxin toxicity Wide complexes Pulseless: immediate defibrillation Unstable: synchronized cardioversion Stable: procainamide, amiodarone, synchronized cardioversion (refractory) If unsure, manage all wide complex tachycardias as ventricular tachycardia

A 55-year-old man presents to the emergency department with palpitations, nausea, and dizziness for 30 minutes. He is afebrile, his heart rate is 140 bpm, and his blood pressure is 78/50 mm Hg. As the nurse attaches him to the cardiac monitor, you see the rhythm strip shown above. What is the most appropriate treatment? AAdenosine 6 mg IV push BDefibrillation at 360 joules CProcainamide 18 mg/kg infused over 30 minutes DSynchronized cardioversion at 200 joules

Which of these children with hematuria needs admission to the hospital? AA 10-year-old boy with recent sore throat treated with amoxicillin, who has tea-colored urine and normal urine output, whose UA shows microscopic hematuria and proteinuria and blood tests are normal BA 12-year-old girl with lower abdominal pain, dysuria, urgency, frequency, and pink urine whose UA shows microscopic hematuria, > 50 WBC/hpf, + leukocyte esterase, + nitrites, and urine pregnancy test is negative CA 6-year-old boy with nausea, vomiting, bloody diarrhea, and decreased urine output whose UA shows microscopic hematuria and hyaline casts and blood tests show a high WBC count, low hemoglobin, and low platelets DAn 8-year-old girl with diffuse mild edema, who has tea-colored urine and normal urine output, whose UA shows microscopic hematuria and proteinuria and blood tests show a normal WBC count, hemoglobin, and platelets but low protein

A 6-year-old boy with nausea, vomiting, bloody diarrhea, and decreased urine output whose UA shows microscopic hematuria and hyaline casts and blood tests show a high WBC count, low hemoglobin, and low platelets hemolytic uremic syndrome (HUS), a disorder that causes a triad of acute renal failure, thrombocytopenia, and microangiopathic hemolytic anemia. It most often results from antibiotic treatment of gastroenteritis in individuals with Escherichia coli O157:H7 infection, -------------------------------------------------------------- vs Poststreptococcal glomerulonephritis (A) results from immune complex deposits in the glomeruli after acute streptococcal infection. Asymptomatic patients with a normal blood pressure may be discharged in consultation with a pediatric nephrologist. Older children with simple cystitis (B) can be safely managed as an outpatient with oral antibiotics. Nephrotic-nephritic syndrome (D) is caused by increased permeability of the glomerular capillary wall leading to renal protein loss and its associated consequences. The hallmarks are proteinuria, hypoproteinemia, generalized edema, and hypercholesterolemia. Patients with nephrotic syndrome with only mild or moderate edema without respiratory symptoms, signs of infection, or thrombotic complications may be discharged home in consultation with a pediatric nephrologist. The treatment of nephrotic syndrome is oral corticosteroids.

Peripheral vasodilation dx: Orthostatic Hypotension Peripheral vasodilationcan be caused by many medications, including terazosin, which is an alpha-1 adrenergic blocker that is used to treat benign prostatic hyperplasia. This class of medication selectively blocks the vascular postsynaptic alpha-1 adrenergic receptors, which reduces peripheral vascular resistance by inhibiting smooth muscle contraction. The resulting peripheral vasodilation is beneficial for the treatment of benign prostatic hyperplasia and hypertension but can often result in orthostatic hypotension as well. ------------------------------------------------------------------- vs Chronic orthostatic intolerance (A) is symptomatically the opposite of orthostatic hypotension and is used to describe dizziness or syncope that occurs when standing for prolonged periods of time. Dizziness caused by orthostatic hypotension occurs when a patient is changing positions, such as from a supine to standing position. Extracellular fluid volume depletion (B) is a cause of orthostatic hypotension but is not consistent with this patient's history as it results from the loss of water and sodium due to a multitude of causes. These could

A 65-year-old man presents to the clinic for intermittent dizziness that has been present for about 3-4 weeks with gradual onset. He reports that he notices it the most when he wakes up in the morning and gets out of bed, and it does go away after a few minutes. He has occasionally noticed dizziness after watching television. When questioned further, he reports that it primarily occurs when he is going from a lying or sitting to a standing position. He reports no syncope or falls. His medical history is significant for benign prostatic hyperplasia and hypertriglyceridemia. He is taking terazosin 4 mg and a fish oil supplement. His vitals on arrival were blood pressure 120/80 mm Hg and pulse 82 bpm. The nurse immediately performed orthostatic vital signs and ECG. The ECG is shown above. Orthostatic vital signs show a supine blood pressure of 130/86 mm Hg and pulse of 75 bpm. Sitting blood pressure is 125/80 mm Hg and pulse is 80 bpm. Standing blood pressure is 110/74 mm Hg and pulse is 89 bpm. Which of the following is the most likely mechanism that explains this patient's dizziness? AChronic orthostatic intolerance BExtracellular fluid volume depletion CImpaired cardiac output DIncreased systemic vascular resistance EPeripheral vasodilation

Which organization is responsible for defining the standards for physician assistant education? AAccreditation Review Commission on Education for the Physician Assistant BAmerican Academy of Physician Associates CCommission on Accreditation of Allied Health Education Programs DNational Commission on Certification of Physician Assistants EPhysician Assistant Education Association

Accreditation Review Commission on Education for the Physician Assistant

A 15-year-old girl presents after a suicide attempt. She reports taking a bottle of over-the-counter pills about 12 hours ago but does not remember the name of the drug. Her symptoms include nausea, vomiting, and diaphoresis. Initial lab work reveals mildly elevated AST and ALT. What drug did she most likely ingest? AAcetaminophen BDiphenhydramine CIbuprofen DPseudoephedrine

Acetaminophen Toxicity Sx: abdominal pain, nausea, vomiting, and jaundice PE: RUQ tenderness Labs: elevated AST and ALT Treatment is N-acetylcysteine (restores glutathione) Rumack-Matthew nomogram: stratifies the risk of liver failure Above Rumack-Matthew nomogram line: treat Below Rumack-Matthew nomogram line: no treatment necessary Determine if overdose was accidental or intentional

A 23-year-old man with a history of type 1 diabetes presents to the emergency department with abdominal pain, vomiting, and fatigue. He has not been taking his insulin. He is found to have diabetic ketoacidosis. His vital signs and mental status are normal except for RR of 26/min and HR of 112 bpm. He has a normal electrocardiogram. Treatment is initiated with intravenous normal saline and insulin. After 1 hour of insulin therapy, his pH is 7.21, glucose is 190 mg/dL, anion gap is 22 mEq/L, bicarbonate is 13 mmol/L, and potassium is 5.5 mEq/L. Which of the following is the most appropriate next step? AAdd dextrose to the intravenous fluids BAdd sodium bicarbonate to the intravenous fluids CStop the intravenous insulin DTreat the hyperkalemia

Add dextrose to the intravenous fluids Treat precipitating cause Correct volume depletion with NS, add dextrose to fluids once glucose is < 200 mg/dL Replete K+ deficit (usually falsely elevated), do not start insulin if K+ < 3.3 mEq/L IV insulin drip until anion gap closes Diabetes-Related Ketoacidosis Patient will have diabetes History of infection, ischemia (cardiac, mesenteric), iatrogenic (e.g. steroids), insulin deficit (poor control), intoxication/illegal (cocaine abuse) (five I's) Abdominal pain, vomiting, and fatigue PE will show fruity-smelling breath, dehydration, and AMS Labs will show hyperglycemia, ketonemia, and an anion gap metabolic acidosis Corrected sodium: add 1.6 mEq/L for each 100 mg/dL in serum glucose HHS = hyperglycemic hyperosmolar syndrome

A 70-year-old woman with a long history of coronary artery disease is seen in cardiology clinic for routine follow up. She complains of continued angina despite medical therapy. She reports chest pain every time she walks to her mailbox. It does not occur at rest and is relieved by sublingual nitroglycerin. Her medications include carvedilol, amlodipine, daytime transdermal nitroglycerine, sublingual nitroglycerin, aspirin, and simvastatin. On physical exam her BP is 105/72, HR is 51 and RR 16. What is the next step in management? AAdd ranolazine BIncrease the dose of carvedilol CIncrease the dose of simvastatin DSchedule 24 hour usage of transdermal nitroglycerin

Add ranolazine should be considered in patients with chronic stable angina who remain symptomaticdespite optimal doses of beta blockers, calcium channel blockers and nitrates.

A 17-year-old G1P0 adolescent at 25 weeks gestation presents with intermittent blurred vision. On presentation, she is currently asymptomatic. Vital signs are HR 84 bpm, BP 165/97 mm Hg, and oxygen saturation 97%. Physical examination reveals 2+ pitting edema on both lower extremities and urinalysis has 3+ protein on dip. Which of the following is the best next step in management? AAdministration of phenytoin BAdmit for further obstetric evaluation CArrange follow-up with the patient's obstetrician DEmergency cesarean section

Admit for further obstetric evaluation dx:severe preeclampsia and should be admitted for further obstetric evaluation. Preeclampsia is defined as gestational hypertension (> 140/90 mm Hg) with proteinuria (> 300 mg/24 hr) that occurs after 20 weeks gestation. Preeclampsia Pregnancy > 20 weeks gestation or postpartum Visual disturbances, severe headaches, or asymptomatic Evaluation will show new-onset hypertension (≥ 140/90 mm Hg) with either proteinuria (≥ 300 mg/24 hr or urine protein: creatinine ratio ≥ 0.3) OR significant end-organ dysfunction Treatment: delivery at 37 weeks (without severe features) and 34 weeks (with severe features) AND prevention of seizures with magnesium sulfate and prevention of permanent maternal organ damage New-onset hypertension < 20 weeks gestation: suspect molar pregnancy

Which of the following scenarios is most consistent with an acute simple febrile seizure? AA 2-month old infant with a rectal temperature of 39.5°C who presents after having two generalized tonic-clonic seizures BA 4-year old child with an oral temperature of 37.8°C who presents after having a generalized tonic-clonic seizure CAn 8-month old infant with a rectal temperature of 39°C who presents after having a generalized tonic-clonic seizure lasting 5 minutes DAn 8-year old child with an oral temperature of 38.8 °C who experienced 10 minutes of right upper extremity and facial twitching followed by brief confusio

An 8-month old infant with a rectal temperature of 39°C who presents after having a generalized tonic-clonic seizure lasting 5 minutes Febrile Seizure - Simple Common in children aged 6 months to 5 years History of fever > 38°C (especially sharp elevation in temp) Single tonic-clonic seizure lasting < 15 mins Treatment is supportive careIf lasting > 5 min, a dose of diazepam gel, nasal, buccal, or suppository can be used

A 45-year-old man with a history of hyperkyphosis that started in his thirties, chronic back pain and stiffness and sacroiliac pain presents to your office as a new patient for his annual exam. He has decreased chest expansion on inspiration and complains of peripheral arthritis in addition to his back pain. Today he complains of acute blurry vision in his left eye with concomitant pain and photophobia. This is his third episode of this eye complaint, though last time it was in the right eye. Which of the following is the most likely diagnosis? AAnkylosing spondylitis BFibromyalgia COsteoarthritis DPolymyalgia rheumatica

Ankylosing spondylitis Ankylosing Spondylitis (Radiographic Axial Spondyloarthritis) Risk factors: male sex, age < 40 Sx: low back pain that's most severe at night and morning stiffness that improves with exercise PE: limited spinal mobility, decreased lumbar lordosis X-ray: squared vertebral bodies, multiple vertebral fusions (bamboo spine) Labs: increased ESR, positive HLA-B27 Treatment options include NSAIDs, physical therapy, TNF-alpha blockers Associated with: uveitis, aortitis, IBD, psoriasis, apical pulmonary fibrosis Diseases associated with HLA-B27: PAIR Psoriatic arthritisAnkylosing spondylitisInflammatory bowel diseaseReactive arthritis

A 20-year-old right-handed man presents to the emergency department after he injured his right shoulder during a basketball game. He was jumping up for a shot when another player collided with him, pushing his right arm over his head in abduction and external rotation. He had immediate pain and fell to the floor. He reports severe pain in the right upper extremity. On physical examination, he holds the arm in external rotation and resists attempts to move it. The contour of his shoulder appears abnormal, with a prominent acromion and loss of the normal rounded shape. He has decreased sensation to light touch over the lateral upper arm. Sensation and motor function are intact over his fingers. Which of the following injuries is most likely responsible for the patient's physical examination findings? AAcromioclavicular joint separation BAcute rotator cuff tear CAnterior glenohumeral joint dislocation DFracture of the surgical neck of the humerus ESpiral fracture of the midshaft of the humerus

Anterior glenohumeral joint dislocation ----------------------------------------------------------------- vs An acromioclavicular joint separation (A) typically results from an impact to the top of the shoulder that causes tearing of the acromioclavicular and sometimes the coracoclavicular ligaments. Physical findings include tenderness over the joint and a prominent distal clavicle.

An 80-year-old woman presents to the clinic reporting chest pain and dyspnea on exertion. Her symptoms have been present for about a year but seem to be worsening lately. She takes lisinopril 20 mg daily for hypertension, acetaminophen 500 mg three times daily for osteoarthritis, and calcium 600 mg daily for her bone health. Her vital signs include a blood pressure of 160/60 mm Hg, heart rate of 80 bpm, respirations of 16/minute, oxygen saturation of 94% on room air, and a temperature of 98.5°F. Physical exam reveals a rapid radial pulse with prominent upstroke and descent, a laterally displaced hyperdynamic apical pulse, a diastolic decrescendo murmur that is high-pitched and loudest at the right sternal border, 2+ pitting edema of the bilateral lower extremities, and crackles at her bilateral lung bases. Which of the following is the most likely pathophysiologic cause of this patient's cardiac condition? AAortic root dilation BChordae tendineae degeneration CCongenital tricuspid mitral valve DPulmonary hypertension ERheumatic heart disease

Aortic root dilation Risk factors: bicuspid aortic valve, rheumatic heart disease, infectious endocarditis, Marfan syndrome Sx: long asymptomatic phase followed by HF symptoms: dyspnea, orthopnea, paroxysmal nocturnal dyspnea, fatigue Widening pulse pressure Bounding water-hammer peripheral pulses Head bobbing with systole (de Musset sign) Prominent nail pulsations (Quincke pulse) Hyperdynamic apical pulse displaced to the left Diastolic blowing murmur best heard along LSB Systolic or diastolic thrill or murmur heard over the femoral arteries (Duroziez sign) In severe AR, a mid-diastolic murmur (Austin-Flint murmur) Diagnosis is made by echo to determine the severity of regurgitation Most commonly caused by abnormal leaflets or proximal aortic root Tx: valve replacement surgery for symptomatic patients and asymptomatic patients with EF < 55% or enlarged LV, or progressive LV changesLimited role of pharmacologic therapy in patients preparing for surgery or non-surgical candidates - diuretics, ACEI, ARB, CCB (amlodipine)

A 65-year-old man presents to his physician for a gradual decline in exertional fortitude and dyspnea on exertion that has been getting worse over the past two months. Physical exam demonstrates a mid-systolic murmur heard best at the second intercostal space near the right sternal border. The murmur decreases with isometric handgrip and Valsalva maneuvers. What is the most likely diagnosis? AAortic regurgitation BAortic stenosis CMitral regurgitation DMitral stenosis

Aortic stenosis

A 10-year-old boy presents with lethargy and vomiting. His parent states the patient had 3 days of cough, rhinorrhea, sore throat, and fever, during which time the nanny had been giving the patient an effervescent over-the-counter medicine to help with his upset stomach. He seemed to be doing much better for a few days until this morning when he became confused and agitated, and he vomited. The physical exam is remarkable for lethargy, mild icterus, and hepatomegaly. Laboratory results include markedly elevated transaminase. Which medication is most likely responsible for this patient's presentation? AAcetaminophen BAspirin CGuaifenesin DIbuprofen

Aspirin Reye syndrome. Reye syndrome is an uncommon, rapidly progressive,noninflammatory encephalopathy associated with altered mental status, cerebral edema, and liver dysfunction. Symptoms typically begin 3-5 days after an apparent recovery from a viral illness. Clinically, patients present with a rapidly progressive encephalopathic picture marked by behavioral changes, a deteriorating level of consciousness, and vomiting. It is a multisystem disease, but the mechanism of injury is not fully elucidated. Patient will be a child History of viral illness treated with aspirin Encephalopathy, delirium, seizures, vomiting PE will show fatty liver Labs will show hypoglycemia, elevated liver enzymes, hyperammonemia, metabolic acidosis Treatment is supportive

Which of the following is an indication for permanent pacemaker placement? AAsymptomatic Mobitz type I second-degree AV block BAsymptomatic Mobitz type II second degree heart block CAsymptomatic sinus bradycardia with heart rate of 40/min DAsymptomatic three second sinus pauses

Asymptomatic Mobitz type II second-degree AV block is an indication for pacemaker placement. This block has a high risk of progressing to complete heart block

What is the most commonly seen symptom or sign in patients with acute aortic dissection? AAortic insufficiency murmur BChest pain CPulse deficit DSyncope

Chest pain Aortic Dissection Risk factors: advancing age, male sex, HTN, Marfan syndrome Sx: acute onset of "ripping" or "tearing" chest pain or back pain PE: asymmetric pulses or SBP difference of > 20 mmHg CXR: widened mediastinum Dx: CT angiography or transesophageal echocardiogram (TEE) Treatment: reduce BP and HR (beta-blockers), pain control, emergency surgery (Type A dissection) Type A: involves ascending aorta Type B: involves only descending aorta

A 3-day-old neonate is brought to clinic with copious eye discharge and eyelid swelling bilaterally. Her mother received minimal prenatal care and delivered the neonate at home. Which of the following is indicated? ACeftriaxone intravenous BGonococcal PCR of eye discharge COral erythromycin DReassurance with outpatient ophthalmologic follow-up

Ceftriaxone intravenous dx:ophthalmia neonatorum, neonatal conjunctivitis caused by Neisseria gonorrhoeae. N.gonorrhoeae is typically acquired during vaginal delivery begins in the first two to five days after birth ---------------------------------------------------------- Oral erythromycin (C) is the treatment of choice for neonatal conjunctivitis or pneumonia caused by Chlamydia trachomatis. Chlamydial conjunctivitis typically begins five to fourteen days after delivery and, in comparison to ophthalmia neonatorum, has a less dramatic onset.

Which of the following medications is most appropriate to use in a 64-year-old man with glucose-6-phosphate dehydrogenase deficiency and a recently diagnosed urinary tract infection? ACephalexin BCiprofloxacin CNitrofurantoin DPhenazopyridine

Cephalexin G6PD Deficiency History of taking antimalarials, sulfonylureas, quinolones, nitrofurantoin, fava beansInfection is also a cause for the hemolysis Labs will show Heinz bodies, presence of bite cells on the smear Consider testing prior to starting potential agents in patients who may be at risk X-linked recessive ---------------------------------------------------------------- vs ciprofloxacin (B), nitrofurantoin (C), and phenazopyridine (D) —are all medications used in the treatment of cystitis. Unfortunately, they are also among the most common causes of hemolytic crises in patients with G6PD. Therefore, they would not be good choices for the treatment of cystitis in this patient.

A 44-year-old man presents with insidious and atraumatic proximal lower leg pain. It is more tender during rest than exercise. It commonly occurs at night, and doesn't seem to improve with oral analgesics. Knee testing is negative for meniscal or ligamentous injury. A screening radiograph reveals no fracture, but does show a non-speckled calcified lesion within the proximal tibia. A bone scan shows no spine or pelvic lytic lesions. Which of the following is the most likely diagnosis? AChondrosarcoma BEnchondroma CMultiple myeloma DOsteoid osteoma

Chondrosarcoma Chondrosarcoma Patient will be 30-50 years old Constant deep achy pain, especially in the nighttime hours, not relieved by rest X-ray (initial imaging) will show lytic lesions or intralesional calcifications (popcorn calcifications) Diagnosis is confirmed with fine-needle aspiration or core biopsy showing malignant chondrocytes in gelatious cartilaginous matrix TreatmentLow-grade tumors: intralesional curettageIntermediate and high-grade tumors: wide en bloc local excision Third most common primary malignancy of bone

A 21-year-old woman with no prenatal care presents for evaluation of lower abdominal pain and fever. She estimates that she is approximately 7.5 months pregnant. On questioning, she acknowledges intermittent pain for two days and a gush of fluid shortly after the pain began. Her temperature is 101.8°F. Physical examination is notable for purulent material in the vaginal vault. Which of the following is the most likely diagnosis? AChorioamnionitis BEndometritis CPelvic inflammatory disease DUrinary tract infection

Chorioamnionitis Intra-amniotic Infection (Chorioamnionitis) Infection, inflammation, or both of the amniotic fluid, placenta, fetus, fetal membranes, or decidua Risk factors: nulliparity, prolonged rupture of membranes, meconium-stained amniotic fluid, internal fetal or uterine contraction monitoring Genital tract infection: STIs, group B Streptococcus, bacterial vaginosis Rx: ampicillin + gentamicin

A 55-year-old man with type II diabetes mellitus, hypertension, and stage I chronic kidney disease presents for vision concerns. He describes dark "strings" in his vision and a general decrease in visual acuity. A funduscopic exam is completed and reveals blot hemorrhages, retinal vascular microaneurysms, and cotton-wool spots. What is the most likely pathogenesis of these findings? AAccumulation of fluid between the neurosensory retina and choroid BBlockage of a retinal vein at an arteriovenous crossing CChronic hyperglycemia-induced retinal injury DRetinal artery occlusion from atherosclerotic disease EVascular injury from increased intraluminal pressure

Chronic hyperglycemia-induced retinal injury Diabetic retinopathy is the leading preventable cause of blindness in adults between the ages of 20 and 74 in the United States.

A 40-year-old G3P3A0 woman presents to the OB/GYN office for cervical cytology screening results. The results of her previous cervical cytology have been unremarkable. She has a history of diabetes mellitus type 2 and takes metformin 850 mg twice per day. Her vital signs are a BP of 131/81 mm Hg, HR of 81 bpm, RR of 16/min, SpO2 of 98% on room air, and a T of 98.7°F. Upon physical exam, normally developed genitalia are seen with no external lesions or eruptions. The vagina and cervix show no lesions, inflammation, or discharge. Her cervical cytology results show atypical glandular cells. What is the best next step in management? AColposcopy with endometrial biopsy BEndometrial and endocervical sampling without colposcopy CHuman papillomavirus testing DLoop electrosurgical excision procedure EReturn to routine Papanicolaou screening

Colposcopy with endometrial biopsy look at chart in pance pearls best next step in evaluating patients ≥ 35 years of age with atypical glandular cells found on cervical cytology.

A patient presents with acute dyspnea and pleuritic chest pain. You suspect pulmonary embolism. Which of the following is the most appropriate test to confirm the diagnosis of a pulmonary embolus? AComputed tomography angiography BD-dimer assay CEchocardiography DVentilation-perfusion scan

Computed tomography angiography ---------------------------------------------------------- vs D-dimer assays (B) for the diagnosis of PE have been extensively studied. They are best characterized as having good sensitivity and negative predictive value, but poor specificity and positive predictive value. Low D-dimer levels appear best at excluding, not including, the diagnosis of PE. Echocardiography (C) reveals PE-suggestive abnormalities in only 30-40% of patients with actual PE. V/Q scanning (D), although counterintuitive, generates data that is insufficient to exclude or confirm the diagnosis of PE.

A 13-year-old girl presents to the clinic with her parents who report small stature and no signs of puberty. She takes no medications, has no prior illnesses, and is up-to-date on her vaccinations. Family history reveals her father is 72 in tall and her mother is 64 in tall and had menarche at age 16. The patient's vital signs are within normal limits, her body mass index is 15 kg/m2, and her height is 55 in. Physical exam reveals a healthy looking girl with no breast buds or pubic hair. Several laboratory results are listed below. Thyroid-stimulating hormone: 1.5 mIU/L Free T4: 0.8 ng/dL (0.8-1.9 ng/dL) Luteinizing hormone: 3 IU/L (0.1-13.4) Follicle-stimulating hormone: 4 IU/L (< 0.1-12) Hemoglobin A1C: 5.0% Prolactin: 2 ng/ml Estradiol: 20 pmol/L Growth hormone: 12 nmol/L ( < 1-14.4) Which of the following is the most likely diagnosis? AConstitutional delay of puberty BFamilial short stature CHyperprolactinemia DHypogonadotropic hypogonadism EHypothyroidism

Constitutional delay of puberty . Puberty is considered delayed when there are no signs of breast development by age 13 in a girl or no signs of testicular enlargement in a boy by age 14. Constitutional delay of puberty refers to puberty that is delayed in the absence of other pathology and that subsequently and spontaneously resumes progress at a later age. The most common cause of constitutional delay of puberty is a family history of delayed puberty.

A 35-year-old man with a history of gastroesophageal reflux disease presents to the emergency department with chest pain and fever. An outpatient esophagogastroduodenoscopy was performed 3 days ago and revealed no gastric ulcers. On exam, he is ill-appearing and tachycardic. Which of the following is the best next step in management? AArrange for repeat emergent esophagogastroduodenoscopy BConsult thoracic surgery COrder a barium esophagogram DPlace a nasogastric tube

Consult thoracic surgery dx: esophageal perforation, Mediastinitis Causes: esophageal rupture or perforation > surgery, head or neck infections CP radiating to neck or intrascapular region Neck or chest crepitus CT Rx: Broad-spectrum Abx, ENT or cardiothoracic surgery consultation

A 23-year-old childcare worker presents to the hospital in labor. She has had no prenatal health care because she has felt well, except for one mild cold. She does not know her due date. The 6.0 lb, 18 in neonate has Apgar scores of 5 and 7 at 1 and 5 minutes, respectively. Physical exam shows microcephaly and splenomegaly. Skin is slightly jaundiced but with no vesicles or erosions. A seizure prompts a CT of the head, which shows a solid ring of solid calcification encircling the two lateral ventricles. What is the most likely causative agent for these newborn findings? ACytomegalovirus BHerpes simplex virus CListeria monocytogenes DParvovirus B19 EToxoplasma gondii

Cytomegalovirus This mother's job as a childcare worker predisposes her to such infection through contact with saliva and other body fluids of infants. This infant's microcephaly, seizure, and periventricular calcifications shown on CT are among the disease's most common neurological manifestations. The periventricular calcifications can be also seen on prenatal ultrasound. Nonspecific evidence includes splenomegaly, hepatomegaly, and jaundice. Neonates with congenital CMV may pass the universal hearing screen at birth, only to have sensorineural hearing loss months later when hearing becomes more critical to the child's development. Their eyes may also be affected with chorioretinitis. They will be born small for gestational age due to intrauterine growth restriction. Some cases will exhibit purpuric papules and macules known as blueberry muffin rash. CMV can be diagnosed with real-time PCR of a saliva swab, and this may become a universal screening test at birth. Treatment is with intravenous ganciclovir or oral valganciclovir, the prodrug. Although treatment carries risks, they are not as great as the risks of nontreatment. ----------------------------------------------------------------- vs Toxoplasma gondii (E), the fungal cause of toxoplasmosis, is also a TORCH disease. It could affect the neonate's central nervous system and produce small, focal intracranial calcifications and seizures, but it would not produce consolidated calcifications around the ventricles, as described in the vignette. Most cases are asymptomatic at birth, with retinal lesions and progressive blindness the most common late manifestations.

A 2-year-old girl presents to the emergency department at 6 pm with bloody vomiting, diarrhea and abdominal pain. Her pregnant mother reports that the patient's grandfather was watching the patient in the afternoon while the mother was shopping. The mother returned home at around 5pm to find her daughter vomiting and with diarrhea and abdominal pain. An abdominal X-ray shows radiopaque particles in the stomach. After fluid resuscitation, what is the next best step in management? ABowel irrigation with polyethylene glycol BDeferoxamine administration CIpecac-induced emesis DN-acetylcysteine administration

Deferoxamine administration dx: Iron Toxicity Inquire about the number of vomiting episodes Patients with myelodysplasia, sickle cell, thalassemia more prone Patients without vomiting in the first 6 hours after exposure will not have significant toxicity GI necrosis, hemorrhage Uncoupling of oxidative phosphorylation → anaerobic metabolism Anion gap metabolic acidosis Stages 1 (< 6 hrs): GI Sx 2 (6-24 hrs): GI Sx improve 3 (12-24 hrs): coma, shock, seizures 4 (2-3 days): hepatic failure, hypoglycemia, coagulopathy 5 (2-8 wks): GI Sx, pyloric scarring, obstruction AXR: most pills radiopaque Whole bowel irrigation for large acute ingestion Chelating agents: deferasirox, deferoxamine, deferiprone Indications: serum Fe level > 500 mcg/dL or severe signs and symptoms (such as metabolic acidosis, repetitive vomiting, toxic appearance, lethargy, hypotension, or signs of shock)

A 25-year-old man presents to your office with rhinorrhea and a sore throat that you believe to be viral in origin. He respectfully requests treatment with antibiotics, and he demonstrates an understanding of the risks, benefits, and alternatives to treatment. His mental status is intact, and you believe him to have full decision-making capacity. Which of the following is the best course of action? ADeny the patient's request BPrescribe amoxicillin CPrescribe ciprofloxacin DRefer the patient to an infectious disease specialist

Deny the patient's request Ethical Principles Autonomy: respect patient's right to decide on treatment options Beneficence: act in patient's best interest Nonmaleficence: refrain from actions that cause harm Justice: resource allocation to maximize benefits and minimize burdens for all patients, regardless of race, socioeconomic class, etc.

A 5-year-old boy with von Willebrand disease presents with persistent bleeding after he lost a tooth four hours ago. Vital signs are unremarkable. Physical exam reveals oozing at the site of the tooth which persists despite pressure. Which of the following is the most appropriate treatment? ADesmopressin BFactor VIII concentrate CFresh frozen plasma DVitamin K

Desmopressin dxvon Willebrand Disease Most common inherited bleeding disorder due to reduced, dysfunctional, or absent von Willebrand factor Sx: increased mucocutaneous bleeding, heavy menses, excessive postpartum bleeding Labs: aPTT may be prolonged (if factor VIII low), VWF antigen, platelet-dependent VWF activity (ristocetin cofactor assay), factor VIII activity Tx options: desmopressin (DDAVP), von Willebrand factor concentrate Most cases autosomal dominant, consider genetic counseling and testing of 1st degree relatives ---------------------------------------------------------- vs Fresh frozen plasma (C) is a product from centrifuged whole blood which contains factors II, V, VII, IX, X, and XI. It should only be used in severe bleeding if factor VIII concentrate is not available.

A 76-year-old man with a 30-year pack history of smoking tobacco presents with a chief complaint of increasing dyspnea on exertion over the past 7 months. What aspect of his history and physical examination would suggest idiopathic pulmonary fibrosis, rather than another diagnosis, as the source of his dyspnea? AChest pain BDigital clubbing CJoint pain DProductive coughing

Digital clubbing dx: Idiopathic Pulmonary Fibrosis Risk factors: male sex, age > 60, history of smoking or environmental exposure Sx: gradual onset dyspnea and dry cough PE: velcro like crackles, clubbing in advanced disease High resolution CT: bibasilar reticular opacities, honeycombing, traction bronchiectasis PFT: restrictive pattern: ↓FVC, ↓TLC and normal or increased FEV1/FVC ratio, ↓DLCO Management includes O2, pulmonary rehabilitation, antifibrotic therapy (pirfenidone and nintedanib), possible lung transplant

A 54-year-old woman presents to discuss options for managing her chronic urticaria. She has tried a broad range of elimination approaches, including avoidance of hot showers, cold, pressure, and multiple suspected allergens. She has also tried a number of antihistamine combinations with minimal results. Which of the following medications would be most promising for this patient? AAspirin BCodeine CDoxepin DOmeprazole

Doxepin a tricyclic antidepressant, is approved and can be very effective for managing chronic urticaria that is refractive to treatment with antihistamines. Urticaria PE: transient blanching, edematous papules, and plaques Etiology: IgE-mediated reaction to an allergen Treatment: supportive care, antihistamines, glucocorticoids (if associated with angioedema) H2-blockers, such as ranitidine or famotidine, which are also used in treated gastroesophageal reflux, may show benefit.

A 51-year-old man with a history of diabetes mellitus and hypertension presents with approximately 60 minutes of left-sided jaw pain that began while he was shoveling snow. Which of the following tests should be ordered? AComputed tomography scan of the maxillofacial region BElectrocardiogram CErythrocyte sedimentation rate DPanorex X-rays

Electrocardiogram Acute Coronary Syndrome: Management Reperfusion PCI - increased survival, decreased ICH and recurrent MI PCI center: < 90 minutes "door to device" time Non-PCI center: transfer for PCI if "door to device" time can be <120 minutes Thrombolysis - if PCI is not available or "door to device" time > 120 minutes Administer within 30 minutes, can be given up to 12 hours of symptom onset Contraindications: ICH, intracranial malignancy, stroke within 3 months, aortic dissection Medical Therapy Oxygen - indicated for O2 sat < 90%, dyspnea, heart failure Nitroglycerin - relieves ongoing chest pain, lowers BP Morphine - relieves pain, reduces work of breathing in setting of pulmonary edemaonly used if nitroglycerin fails to relieve the paincan be associated with negative outcome Beta Blockers - prevent recurrent ischemia and dysrhythmia Antiplatelet agents - reduce recurrent coronary artery thrombosis, stent thrombosis, and deathAspirin - given before PCI and continued indefinitelyP2Y12 inhibitor (clopidogrel, prasugrel, ticagrelor) - continue for 1 year if stent placedGPIIb/IIIa antagonists for patients undergoing PCI Anticoagulation - if LV thrombus or Afib present and all patients receiving thrombolytic therapy ACE Inhibitors - reduce cardiovascular events, prevent LV remodeling High dose statin - lipid lowering, lowers risk of death, recurrent MI, and stroke

A 72-year-old woman in the intensive care unit begins to show signs of decreased ventilatory drive and critical oxygen desaturation. After initiating mechanical ventilation via endotracheal intubation under moderate sedation, her vital signs stabilize. Her chest radiograph after 72 hours has progressive bilateral pulmonary infiltrates and alveolar infiltrates. Vital signs at this time include HR of 110 bpm, RR of 21/minute, BP of 121/96 mm Hg, T of 98.1°F, and SpO2 of 85% on previously appropriate mechanical ventilation settings. Which of the following measures improves outcomes? AEarly parenteral nutrition BElevate the head of the bed to 30° CProne positioning DProphylactic antibiotics

Elevate the head of the bed to 30°

Which of the following is true regarding botulism? AFoodborne botulism develops 10-14 days following toxin ingestion BFoodborne botulism is caused by ingestion of a heat-stable toxin (type A) CInfantile botulism is the most common form DIntestinal colonization is common in foodborne botulism

Infantile botulism is the most common form

55-year-old woman presents to the oncology clinic to discuss therapy options for her newly diagnosed, non-metastatic, unilateral, invasive breast cancer. Routine analysis reveals estrogen receptor-positive and progesterone receptor-positive tumor expression. Which of the following is the recommended therapy for this patient? ACytotoxic chemotherapy alone BEndocrine therapy with breast surgery CMastectomy alone DRadiation therapy and radical mastectomy

Endocrine therapy with breast surgery Endocrine therapy,chemotherapy, or biologic therapy are all systemic adjuvant therapy options which can be used to treat breast cancer in addition to surgical therapy and radiation. Tamoxifen, an estrogen receptor modulator, significantly reduces the risk of recurrence and death in patients with ER-positive disease, such as the patient in the vignette. Screening for patients with average risk: USPSTF: biennial screening mammography starting at the age of 50 NCCN: annual screening mammography starting at the age of 40 Breast self-examination should NOT be routinely encouraged --------------------------------------------------------------- vs . Mastectomy alone (C) is the surgical therapy performed when breast-conserving therapy is not indicated or when an affected woman opts for mastectomy instead of breast-conserving therapy. A woman with ER-positive disease may choose mastectomy or breast-conserving therapy in addition to endocrine therapy. The goal of radiation therapy (D) is to eradicate any post-breast-conserving therapy residual cancer. Radical mastectomy would not be recommended in a patient with unilateral, non invasive, non metastatic breast cancer.

A 27-year-old woman presents to her primary care physician with dysphagia. Her symptoms began several months ago. She has trouble swallowing solids and liquids, though liquids seem to make her choke and sputter the most. Therefore, she has been unable to eat and has thus experienced significant weight loss. She has no significant past medical history apart from a 20 pack-year smoking history. She reports no recent travel. Physical examination is within normal limits. A barium esophagram reveals a bird beak appearance. What is the next best step in diagnosis? ABegin a calcium channel blocker BBegin botulinum toxin injections CEsophageal manometry DUpper endoscopy

Esophageal manometry Achalasia Sx: dysphagia to solids and liquidsChronic Chagas disease may present similarly PE: absent peristalsis in the lower esophagus Barium swallow shows bird-beak appearance Diagnosis is confirmed with esophageal manometry: increased LES pressure

A 45-year-old man with thin build and a medical history of hypertension, dyslipidemia, and a 25 pack-year smoking history presents to the clinic for evaluation of his dyspnea. He states he becomes breathless after walking up a slight hill and reports an infrequent cough with clear sputum. Current medications include losartan and atorvastatin. Vital signs include HR 91 bpm, RR 17 breaths per minute, BP 152/86 mm Hg, T 98.1°F, and SpO2 95% on room air. Chest auscultation reveals a quiet chest without adventitious sounds. Which of the following is likely to be found in pulmonary function test results? ADiffusion capacity for carbon monoxide of 120% predicted BFEV1 of 35% of predicted prebronchodilator and 75% of predicted postbronchodilator CFEV1/FVC- of 0.57 DFunctional residual capacity of 65% predicted ETotal lung capacity of 60% predicted

FEV1/FVC- of 0.57 -------------------------------------------------------------------- vs The diffusion capacity for carbon monoxide of 120% predicted (A) is an obstructive lung disease finding that is associated with airway narrowing, such as in asthma. An FEV1 of 35% of predicted prebronchodilator and 75% of predicted postbronchodilator (B) is indicative of a reversible obstructive airway disorder, most commonly asthma. Functional residual capacity of 65% predicted (D) is demonstrated with restrictive lung diseases. More specifically, this finding often coincides with restriction due to chest wall abnormality, such as that seen in moderate obesity. Total lung capacity of 60% predicted (E) is demonstrated with restrictive lung diseases that are due to increased lung elastic recoil, such as pulmonary fibrosis.

An 18-year-old woman presents to the clinic reporting several episodes that she describes as blackouts. She reports feeling a fluttering in her stomach and a sense of anxiety and approaching doom, then does not remember anything for a few seconds to a few minutes. Her mother states when her daughter has these episodes, she stares into space, tilts her head to the left, and smacks her lips repetitively, although the patient has no recollection of this. The patient reports having no prior illness, trauma, or surgery and does not take any medications or illicit drugs. Her vital signs are within normal limits. Her physical exam, including the neurologic exam, is normal. CBC and CMP results are normal, and her urine drug screen is negative. Which of the following is the most likely diagnosis? AAcute dystonia BFocal onset seizure CGeneralized tonic-clonic seizure DPsychogenic nonepileptic seizure ETourette syndrome

Focal onset seizure Common signs and symptoms of focal onset seizures are an aura, automatisms, and dystonic posturing, while uncommon signs and symptoms are affective disordered behavior, vocalizations, or hypermotor behavior. Examples of auras encountered in focal onset seizures include a feeling of impending doom, anxiety, fear, nausea, or deja vu. Examples of automatisms include lip-smacking, chewing, fumbling, picking, or fidgeting. Dystonic posturing usually manifests as head turning. Patients may retain consciousness throughout the episode (focal onset seizure with retainedconsciousness) or may lose awareness (focal onset seizure with impaired awareness). Those patients with impaired awareness have an arrest in their normal behavior and stare off into space for 30 to 120 seconds. Patients may experience postictal confusion, which is usually brief. Definitive diagnosis is with electroencephalography --------------------------------------------------------------- Generalized tonic-clonic seizures (C) involve widespread abnormal electrical activity in the brain and cause loss of consciousness, involuntary whole-body movements in a tonic-clonic pattern, and may also involve atonia and myoclonic jerking.

A 43-year-old woman presents to the primary care office for concerns of sadness. She states that she has had some stress in her family recently. She has noticed, and her partner has mentioned, that her mood seems very labile and she usually seems sad. She struggles to gain interest in going to the gym, which she used to love to do daily. She feels tired most days for "no reason" as she gets 7-8 hours of sleep nightly. She is having a hard time concentrating at work and has noticed she is withdrawing from friendships. After discussion, the patient decides she would like to try medication. You are going to start her on the recommended first-line class of medications for the suspected diagnosis. What are the early side effects of the first-line medication that you should discuss with the patient? AHyperthermia, tachycardia, sedation BInsomnia, anxiety, headache CNausea, dizziness, diaphoresis DSedation, hyperlipidemia, hyperglycemia EWeight gain, dizziness, involuntary muscle twitches

Insomnia, anxiety, headache Selective serotonin reuptake inhibitors (SSRIs)

A 46-year-old man presents with two days of worsening dysphagia. He now is complaining of dyspnea that is worse with a supine position. Physical exam reveals bilateral cervical lymphadenopathy and pain with gentle palpation of the upper trachea. His heart rate is 112 beats/minute, temperature is 102.3°F, and oxygen saturation is 94% on room air. Which of the following findings is most consistent with the suspected diagnosis? A"Steeple sign" on radiography BExpiratory wheezing CInspiratory stridor DTrismus

Inspiratory stridor dx Epiglottitis, Epiglottitis Sx: rapid onset of fever, muffled voice, dysphagia, respiratory distress PE: sore throat, stridor, leaning forward, drooling, red swollen epiglottis on direct visualization Imaging: enlarged epiglottis (thumbprint sign) on lateral neck X-ray Most commonly caused by H. influenzae (decrease since Hib vaccine), Strep and Staph species Treatment is airway management and IV antibiotics (third-generation cephalosporin AND antistaphylococcal)

A 45-year-old man is brought to the emergency department after being the unrestrained passenger in a motor vehicle collision. His vital signs on arrival are T 37.3°C, HR 90, BP 140/94, RR 23. His physical exam is significant for a GCS of E3V4M5 and a large left temporal scalp laceration from hitting the windshield. CT imaging of his head reveals a left temporal linear skull fracture under the area of the laceration with no signs of intracranial hematoma formation. Which of the following therapies is indicated in this patient? AHypertonic saline BIntravenous antibiotics CMannitol DNicardipine

Intravenous antibiotics A linear fracture of the skull with an overlying scalp laceration is considered open. Patients with open skull fractures should receive antibiotic prophylaxis with vancomycin and ceftriaxone. This is also true of patients with skull fractures that are depressed, cross a sinus, or are associated with pneumocephalus

A 47-year-old woman with diabetes mellitus is found to have latent tuberculosis during a routine employee health purified protein derivative test. Which of the following medications in her drug regimen indicate she should receive supplementation with pyridoxine (vitamin B6)? AEthambutol BIsoniazid CPyrazinamide DRifampin

Isoniazid

A 40-year-old man presents with bloody stools for six months. He also reports a severe, tearing pain during defecation followed by throbbing discomfort. Medical history is significant for constipation. He has undergone multiple conservative treatments without improvement of his symptoms. He continues to have painful defecations. A fibrotic skin tag at the outermost edge of the anoderm is noted on physical exam. Which of the following is the most appropriate next step in management?😨😨😨😨 ADiltiazem 2% ointment BLateral internal sphincterotomy CSilver nitrate 2% DSitz baths ETopical hydrocortisone

Lateral internal sphincterotomy dx: Anal fissures are benign lesions that are rocket-shaped or linear and measure less than 5 mm. They are most commonly found in the posterior midline but can also occur anteriorly. Infants and middle-aged individuals are often affected. Anal fissures that do not occur in the midline should raise concerns for Crohn disease, human immunodeficiency virus, tuberculosis, syphilis, or carcinoma. Patient presents with rectal pain and bleeding that occurs with or shortly after defecation PE will show fissure located in the posterior midline Diagnosis is made by visual inspection Treatment is stool softeners, protective ointments, sitz baths, topical nitroglycerin or nifedipine If fissures are located laterally, search for pathologic etiologies What is the most feared complication of lateral internal sphincterotomy? Answer: Incontinence. Chronic fissures refractoryto medical therapy may benefit from lateral internal sphincterotomy. The purpose of the surgery is to excise the hypertrophied internal sphincter. There are two approaches for lateral internal sphincterotomy: open and closed. Complications of lateral internal sphincterotomy include infection, bleeding, fistula development, and incontinence. -------------------------------------------------------------- vs Sitz baths (D) are considered a conservative measure and may provide temporary pain relief. The patient in the vignette above has failed conservative treatment and requires surgical treatment with lateral internal sphincterotomy.

An 85-year-old nursing home patient presents with diffuse abdominal pain and distension with nausea but no vomiting. The above abdominal radiograph is obtained. What is the most appropriate management? AEnema BNasogastric tube and bowel rest CSigmoidoscopy DSurgical intervention

Sigmoidoscopy dx: Sigmoid Volvulus

An 85-year-old woman presents to the emergency department with the sudden onset of mid-back pain after she bent down to pick up her shoes. She tried taking acetaminophen with minimal relief. The pain is worse with movement, is localized to the back, and does not radiate to the legs. She is otherwise in good health and lives independently. On physical examination, she appears uncomfortable and ambulates with a slow but symmetric gait. Body mass index is 20 kg/m2. She has a mild thoracic kyphosis. She is tender to palpation over the lower thoracic spine in the midline. She has pain and limited mobility with forward bending and extension of the spine. Neurologic examination is normal. Which of the following is the most likely underlying etiology of the patient's symptoms? ADegeneration of intervertebral discs BLoss of bone mineral density in the spine CNarrowing of the spinal canal due to arthritis DOssification of the posterior longitudinal ligament EParaspinal ligament sprain

Loss of bone mineral density in the spine osteoporotic vertebral compression fracture. Osteoporotic fractures are considered fragility fractures, defined as fractures that occur from a fall of a standing height or less, with minimal trauma. Most vertebral compression fractures occur in the mid-thoracic spine or thoracolumbar junction. The pain is typically localized to the spine and does not radiate to the legs. Patients with multiple compression fractures may exhibit an increased thoracic kyphosis (dowager hump) or loss of height. X-rays reveal the fracture, and advanced imaging is rarely necessary. Treatment is pain relief and gradual return to activity

A 62-year-old man who does not smoke and has no significant medical history presents to your office with questions about his cholesterol. Recent laboratory results include total cholesterol 253 mg/dL, high-density lipoprotein cholesterol 38 mg/dL, low-density lipoprotein cholesterol 165 mg/dL, and triglycerides 250 mg/dL. He has a 10-year atherosclerotic cardiovascular disease event risk score of 9%. Which of the following is the most appropriate initial therapy? AFenofibrate BFish oil CGemfibrozil DLovastatin

Lovastatin LDL-C ≥ 190 mg/dL: no risk assessment, high-intensity statin DM and age 40-75 years: moderate intensity statin DM and age 40-75 years: risk assessment to consider high-intensity statin Age > 75 years: clinical assessment, risk discussion ------------------------------------------------------- Fenofibrate (A) and gemfibrozil (C) are used in the treatment of hypertriglyceridemia. Fish oil (B) is recommended for patients with refractory hypertriglyceridemia and is not first-line therapy for hypercholesterolemia.

A patient of yours with a history of diverticulosis develops acute diverticulitis. He is admitted to the hospital for inpatient care. He is successfully treated and discharged home. He follows up with you a few days later. Which of the following should you most likely recommend to him at this time? AColonoscopy within 3 days BHigh fiber diet until 6 weeks of no symptoms CLow fiber diet until 6 weeks of no symptoms DSigmoidoscopy within 3 days

Low fiber diet until 6 weeks of no symptoms because it helps reduce the frequency of stools and allows the affected portion of the colon to adequately heal. Diverticulitis Sx: abdominal pain that is localized to the left lower quadrant, fever, nausea, vomiting, and a change in bowel habits PE: localized guarding, rigidity, and rebound tenderness Diagnosis is made by CT with IV contrast: thickened bowel wall, "fat stranding," may show complications - bowel perforation, abscess, fistula, obstruction Consider treatment with supportive care and/or antibiotics based on risk factors and presentation Antibiotics to cover gram-negative and anaerobic bacteria, bowel rest, and surgery (in severe cases) High-fiber diet can help in prevention ----------------------------------------------------------- vs A high fiber diet (B) is a recommended preventative strategy for patients with diverticulosis. However, high fiber intake is held in the setting of acute diverticulitis until the patient has no symptoms for 6 weeks after resolution.

An older patient presents to the office complaining of lower extremity skin changes and discomfort that is relieved with elevation. Which of the following would most likely be present on physical exam? ACold lower extremities BDiminished pulses CLower extremity edema DPalpable cord

Lower extremity edema dx:Venous Insufficiency Risk factors: obesity, pregnancy, advancing age, prolonged standing, family history of venous insufficiency Sx: leg swelling PE: skin changes, ulceration, edema, varicose veins Imaging: valvular abnormalities Diagnosis is made by history, physical exam Most commonly caused by valvular abnormalities Tx: leg elevation, compression therapy, ulcer care, ablation

Which of the following statements is most accurate regarding health maintenance for a patient with chronic pancreatitis? AAbstinence from alcohol will improve symptoms but will not decrease mortality BEnteric coated pancreatic enzymes require coadministration with an H2 antagonist CLong chain triglycerides can provide extra calories in patients with weight loss DMalabsorption of fat soluble vitamins may occur but is rarely clinically symptomatic

Malabsorption of fat soluble vitamins may occur but is rarely clinically symptomatic

A 27-year-old man with no reported prior medical history presents to the emergency department via EMS after a gunshot to the right side of his chest. His vitals are heart rate 151 bpm, respiratory rate 32 breaths per minute, and blood pressure 88/57 mm Hg. There is noted to be decreased chest wall movement to the affected side, hyper-resonant chest percussion, and minimal to no breath sounds are heard to the right lung. What is the most appropriate initial management for this patient's condition? AComplete a bedside thoracoscopy BObserve the patient and administer supplemental oxygen COrder chest radiograph DPerform needle thoracostomy EPerform thoracentesis

Perform needle thoracostomy dx:tension pneumothorax

A 26-year-old woman who is 16 weeks pregnant and has a history of Graves disease on propylthiouracil presents to the endocrinology clinic for routine follow-up. Vital signs today include a heart rate of 80 bpm, blood pressure of 120/80 mm Hg, respiratory rate of 20/min, oxygen saturation of 98% on room air, and temperature of 98.6°F. Physical examination reveals no lid lag, exophthalmos, tremor, or goiter. Laboratory studies include a thyroid-stimulating hormone level of 2 mU/L and a free thyroxine level of 1 ng/dL. Which complication will be reduced by switching this patient's therapy from propylthiouracil to methimazole? AAgranulocytosis BFetal facial abnormalities CFetal skin abnormalities DMaternal hepatotoxicity EMaternal nephrotoxicity

Maternal hepatotoxicity dx symptomatic hyperthyroidism due to Graves disease, toxic adenoma, toxic multinodular goiter, or gestational trophoblastic disease require therapy for hyperthyroidism during pregnancy. Thionamide therapy is used in pregnant patients to decrease thyroid hormone synthesis. Propylthiouracil is the recommended thionamide if initiating therapy during the first trimester because the associated fetal anomalies are less severe than with methimazole. methimazole is associated with a lower risk of maternal hepatotoxicity ---------------------------------------------------------------- vs Fetal facial abnormalities (B) are more likely to occur with methimazole exposure during pregnancy than propylthiouracil. Because of the risk of facial abnormalities and other fetal anomalies, propylthiouracil is the preferred thionamide during the first trimester of pregnancy.

You are evaluating a 73-year-old woman with a history of weakness, fatigue, and difficulty walking. She lives alone and unassisted. She has no past medical history and takes no medications. Physical examination shows an enlarged, smooth, and tender tongue. Pallor of the conjunctivae is also seen. Which of the following laboratory studies would be most likely to confirm the diagnosis? AIron panel Your Answer BMean corpuscular volume CMethylmalonic acid and homocysteine levels DPeripheral blood smear

Methylmalonic acid and homocysteine levels dx:Vitamin B12 (Cobalamin) Deficiency presents with paresthesias (eg, numbness and tingling), ataxia, glossitis, and megaloblastic anemia (eg, weakness, fatigue, easy bruising). Laboratory studies will show increased serum (and urine) concentrations of both methylmalonic acid (MMA) and homocysteine (HC). -------------------------------------------------------------- vs An iron panel (A) is typically used in diagnosing iron deficiency anemia (microcytic anemia). Mean corpuscular volume (MCV) (B) is a measure of the average volume of a RBC and would only be helpful in confirming microcytic, normocytic, or macrocytic anemia. Since both folate and vitamin B12 deficiency result in macrocytic anemia, this test would not be useful in this situation.

An 18-year-old agricultural laborer presents with an intensely sore throat and fever. He reports he has never been sick before. The patient's vaccination history is not available. Vitals are BP of 110/65 mm Hg, HR of 70 bpm, RR of 12 bpm, T of 100.8°F, and BMI of 20 kg/m2. The submandibular area shows significant edema, and laryngoscopic exam reveals a dull-colored, leathery plaque adhering to the right tonsil, which bleeds upon scraping. What is one of the potential complications of this patient's disease? AEsophagitis BHemolytic uremic syndrome CMyocarditis DReactive arthritis ESplenic rupture

Myocarditis dx: diphtheria, caused by gram-positive Corynebacterium diphtheriae, which can produce a toxin that destroys tissue. Patient presents with URI-like illness, sore throat, low fever PE will show bull neck, cervical adenopathy, pseudomembrane on pharynx, and grains of salt on tonsils Most commonly caused by Corynebacterium diphtheriae (aerobic Gram positive) The bacterium enters the nasopharynx and, if it is a toxin-producing strain, develops a lesion called a pseudomembrane on the nasopharynx or brachial tree. The word diphtheria comes from the Greek word for leather, descriptive of its thickness. Diphtheria lesions are known to bleed when scraped. Infected patients may have a sore throat, barking cough, and hoarseness with a mild fever. Cervical lymphadenopathy and submandibular swelling is common for serious diseas

A 19-year-old man who is employed in construction presents with pain to his thumb and wrist for 4 months. He states the pain worsens while he is working. Examination reveals pain along the radial surface of the wrist with forced ulnar adduction. What management is indicated? AMRI of the wrist BNSAIDs and thumb spica splint CSurgical release of the dorsal extensor compartment DWrist X-ray with ulnar deviation view

NSAIDs and thumb spica splint dx:de Quervain disease

A 21-year-old man presents somnolent and altered. There is no evidence of trauma and his pupils are 2 mm. His vital signs are: T 36.8°C, HR 58, RR 6, BP 96/52, and oxygen saturation 93% on room air. His fingerstick glucose is 85 g/dL. Which of the following is an appropriate treatment? AAtropine BFomepizole CNaloxone DNaltrexone

Naloxone dxOpioid Overdose ------------------------------------------------------------ Fomepizole (B) is the antidote used in toxic alcohol ingestions. By blocking alcohol dehydrogenase, fomepizole prevents the production of the toxic metabolites of ethylene glycol and methanol. Atropine (A) is a pure anticholinergic agent that is used as the antidote in cholinergic poisoning (e.g. insecticide poisoning). Naltrexone (D) is an opioid antagonist that is primarily used for patients with alcohol dependence. Its mechanism of action is not fully understood. It can be used in patients with opioid addiction as well to help block opioid receptors and prevent additional recreational use. It is not used acutely in the reversal of opiate intoxication

A 35-year old man presents to the emergency department with superficial facial lacerations and epistaxis. After treating the epistaxis, you examine the nasal mucosa. Which of the following is considered an emergency, requiring urgent treatment? AA deviated septum BClosed nasal fracture CSeptal hematoma DTrauma to Kiesselbach plexus

Septal hematoma

A 62-year-old man with a history of hypertension and tobacco abuse presents with acute onset of sharp epigastric abdominal pain with radiation to his back. On arrival, his vitals signs are T 37.3°C, HR 100, BP 180/90 in the right arm and 80/40 in the left arm, RR 27. Which of the following agents is the first line management of this patient's condition? ADiuretics BNegative inotropes CVasodilators DVasopressors

Negative inotropes are the preferred agents for the control of hypertension in aortic dissection. This is due to their ability to lower blood pressure without increasing heart rate, which would increase shearing force against the intimal flap and lead to propagation of the dissection. Short-acting beta-blockers, such as labetalol or esmolol, are the first line negative inotrope agents. For persistent hypertension despite maximal therapy with negative inotropes, vasodilators can be used. dxAortic dissection Risk factors: advancing age, male sex, HTN, Marfan syndrome Sx: acute onset of "ripping" or "tearing" chest pain or back pain PE: asymmetric pulses or SBP difference of > 20 mmHg CXR: widened mediastinum Dx: CT angiography or transesophageal echocardiogram (TEE) Treatment: reduce BP and HR (beta-blockers), pain control, emergency surgery (Type A dissection)Type A: involves ascending aortaType B: involves only descending aorta

A 21-year-old woman presents with urinary frequency and dysuria for 2 days. Vital signs are within normal limits. Examination reveals mild suprapubic tenderness with no costovertebral angle tenderness. What is appropriate management of this patient? ACeftriaxone BNitrofurantoin CNoncontrast CT scan of the abdomen and pelvis DPhenazopyridine

Nitrofurantoin dx:uncomplicated acute cystitis

A 2-year-old boy with no significant medical history presents to the primary care clinic for left ear pain and left eye crusting and discharge. For the past 4 days, he has had eye redness, crusting, and discharge from the left eye. His parent reports his left eye appears to be stuck shut in the mornings. His parent also says he has been reporting left ear pain and pulling on the left ear for the past 3 days. He has had a fever of up to 102°F during this time, and it has responded to antipyretics. He has not had nasal congestion, cough, rhinorrhea, vomiting, diarrhea, or a rash. The patient does not take any medications on a regular basis. His heart rate is 94 beats/minute, blood pressure is 100/65 mm Hg, respiratory rate is 20 breaths/minute, oxygen saturation is 100%, and temperature is 98.4°F. His physical exam reveals conjunctival injection with a small amount of purulent yellow discharge from the left eye. The right eye is normal in appearance. On otoscopic examination of the ears, the left tympanic membrane is intact, bulging, and opaque yellow. The right tympanic membrane is intact, translucent, and pearly gray. Which of the following is the most likely causative organism for this

Nontypeable Haemophilus influenzae dx: otitis-conjunctivitis syndrome patients will often report a history of ocular symptoms of conjunctivitis followed by ear pain, hearing changes, or discharge from the ear. Typical symptoms of bacterial conjunctivitis include unilateral conjunctival injection, purulent discharge from the eye, and crusting around the eye. ------------------------------------------------------------------ vs Staphylococcus aureus (E) is a less common cause of acute otitis media in children. S. aureus can often be isolated as a cause for infection in children with tympanostomy tubes.

A young woman with Raynaud phenomenon presents with a three month history of cyclical short episodes of chest pain. Her social history is positive for intermittent cocaine use. The pain occurs most commonly after she wakes in the morning, lasts for 20 minutes, then resolves. This pain occurs at rest and is not worse with exercise or increased activity. Which of the following would you most expect to find during an evaluation of these symptoms? ACoronary artery stenosis BNormal exercise stress test CPain relief during ergonovine administration DPersistent ST elevation

Normal exercise stress test Vasospastic Angina (Prinzmetal Angina) History of HTN, smoking, DM, obesity, or cocaine use Squeezing, pressure-like chest discomfort at rest ECG will show transient ST segment elevations Cardiac enzymes will be normal Diagnosis is made by cardiac stress test Most commonly caused by coronary artery spasm Treatment is calcium channel blockers and nitrates ------------------------------------------------------------------ Patients with vasospastic angina typically have minimal to absent coronary stenosis (A). More pain, not less (C), occurs during administration of ergonovine in a patient with coronary vasospasm. Persistent, diffuse ST elevation (D)would be seen with pericarditis, not vasospastic angina.

72-year-old man presents to the emergency department for disorientation. According to his family, the patient has been progressively more withdrawn, apathetic, and complaining of worsening nausea, vomiting, anorexia, drowsiness, and lethargy during the last few months. His past medical history includes type-2 diabetes controlled with diet and metformin and an uncomplicated appendectomy 15 years ago. Vital signs are within normal range. On physical examination, he is disheveled, disoriented, drowsy, and dehydrated. Laboratory results reveal hemoglobin 13 g/dL, platelets 310,000/µL, sodium 145 mEq/L, potassium 4.9 mEq/L, chloride 101 mEq/L, calcium 14.1 mg/dL, glucose 91 mg/dL, BUN 8 mg/dL , and creatinine 0.8 mg/dL. In addition to subcutaneous calcitonin and a bisphosphonate, which of the following is the most appropriate next step in management? AGlucocorticoids BHemodialysis CHydrochlorothiazide DNormal saline EVitamin D

Normal saline dx: hypercalcemia ECG: shortened QT interval Most common causes Malignancy (most common inpatient cause) Primary hyperparathyroidism (most common outpatient cause) Treatment: IV fluids, bisphosphonates, calcitonin, denosumab (refractory disease)

13-year-old boy presents with left-sided otalgia and pain behind his ear. His ear pain has been worsening over the past week and now he has ringing in his ears. Physical exam reveals post-auricular tenderness, warmth, and swelling with displacement of the pinna. The ear canal is swollen, and the tympanic membrane cannot be visualized. Vital signs include blood pressure of 113/68 mm Hg, pulse of 112 beats per minute, and oral temperature of 38.1°C. Which of the following is the most appropriate next step? ABegin an oral macrolide and topical fluoroquinolone BEmergently consult otolaryngology CIrrigate the external auditory canal in order to visualize the tympanic membrane DOrder a CT scan and begin empiric intravenous antibiotics

Order a CT scan and begin empiric intravenous antibiotics dx: acute mastoiditis. The best initial management is to order a CT scan and begin IV antibiotics that cover the most common causative pathogens, including S. pneumoniae, M. catarrhalis, and H. influenzae.

A 17-year-old girl is examined for a routine visit. She eats a healthy diet. She also stays active by playing volleyball three times a week. Her grades are mostly Bs. She reports she started to be sexually active for the past six months and has delayed periods for two months now. She had her menarche at 12 years old and has regular periods. She also smokes a quarter of a pack of cigarettes per day. Which of the following is an adverse pregnancy outcome due to maternal smoking? AHyperbilirubinemia BLarge for gestational age CPlacental abruption DRespiratory distress syndrome

Placental abruption Cigarette smoking during pregnancy is the most important modifiable risk factor associated with adverse pregnancy outcomes. Smoking and secondhand smoke exposure increase the risk of infertility, placental abruption, preterm premature rupture of membranes (PPROM), and placenta previa.

An otherwise healthy 30-year-old woman presents to the clinic with 4 months of constipation. She has tried taking psyllium with minimal relief. Her physical exam is unremarkable. Other than educating her on increasing exercise, which of the following initial treatments would you recommend? ADisimpaction BLactulose CLinaclotide DPolyethylene glycol

Polyethylene glycol Osmotic laxatives are a good option, such as polyethylene glycol - an increasingly common first-line treatment with robust evidence of benefit. Most outpatient constipation can be relieved with diet changes such as increasing fiber(psyllium or methylcellulose products) intake and decreasing milk, coffee, tea, and alcohol intake. Also, increasing daily exercise can be helpful. Bulk formers, like psyllium, have shown to be very beneficial and cost-effective. If a patient fails this easy option, then one should consider adding or changing to a different class. Osmotic laxatives are a good option, such as polyethylene glycol - an increasingly common first-line treatment with robust evidence of benefit. Stool softeners like docusate and stimulant laxatives like senna or bisacodyl may also be tried. Other options include mineral oil (offers rapid treatment for acute cases) and tegaserod (a prokinetic used in severe cases). Inpatient care, or severe refractory outpatient management, may require transrectal enemas or manual disimpaction. ------------------------------------------------------------- vs Disimpaction (A) is usually necessary for inpatient care of the elderly with significant comorbidities, those with spinal cord injury, and those with traumatic brain injury.

A 14-year-old boy presents to clinic with a one month history of right shoulder pain. The patient is a competitive tennis player. Symptoms began during a match when the patient was serving. He experienced a pop in the shoulder and had immediate pain. Magnetic resonance imaging of the shoulder shows a superior labral tear from anterior to posterior (SLAP lesion). Which of the following is the most likely physical exam finding? APositive cross arm test BPositive Neer test CPositive O'Brien test DPositive sulcus test

Positive O'Brien test

You examine a 12-month old boy in clinic for possible measles exposure. He stayed at his grandmother's house two days ago where he played with his cousin. The mother is concerned because the boy's cousin developed measles that is confirmed by the laboratory. The mother denies any symptoms on the boy. The boy has not had his measles vaccination. Which of the following is the best therapy for the boy? AMeasles immune globulin BMumps, measles, rubella vaccination CRibavirin DVitamin A

Postexposure prophylaxis for susceptible individuals exposed to measles consists of vaccination within 72 hours of exposure in the absence of a contraindication. Mumps, measles, rubella vaccination (MMR) vaccination can provide some protection or modify the clinical course of measles if administered within 72 hours of measles exposure.

An 8-year-old girl is seen in your office with abdominal pain, bloody diarrhea, and weight loss for the past few weeks. Her family just returned from Africa. You obtain a stool sample and find Schistosome eggs. Which of the following medications is the treatment of choice for this infection once the diagnosis is established? AAlbendazole BIvermectin CPraziquantel DPrednisone

Praziquantel Schistosomiasis (Bilharzia) Caused by parasitic worms Contact with contaminated water Prevention by avoiding contaminated water Treat with praziquantel

A 16-year-old boy is in clinic for a routine health check. He is doing well in school and has plans to join the military when he reaches 18 years of age. He practices and plays basketball about 60 minutes per day. He denies smoking and drug use. Physical examination is normal. At this visit, you recommend meningococcal vaccine booster. Which of the following is the best statement regarding meningococcal infection? AChemoprophylaxis can be administered two weeks after exposure to the index case BDroplet precaution is not necessary for patients infected with meningococcal meningitis CNasopharyngeal cultures are needed in determining the need for chemoprophylaxis DProphylaxis is indicated in close contacts of patients with meningococcal infection

Prophylaxis is indicated in close contacts of patients with meningococcal infection ------------------------------------------------------------- droplet precaution is not necessary for patients infected with meningococcal meningitis (B) is false. In fact, droplet precautions should be continued for 24 hours after institution of effective antibiotics in patients with suspected or confirmed Neisseria meningitidis infection. Nasopharyngeal cultures are needed in determining the need for chemoprophylaxis (C) is wrong since it is not necessary to obtain nasopharyngeal cultures before starting chemoprophylaxis.

A 23-year-old G1P0 woman presents to the emergency department with vaginal bleeding. She is 8 weeks and 2 days pregnant according to an outpatient ultrasound she had last week showing an intrauterine pregnancy with a detectable fetal heart rate. She describes the bleeding as lighter than a normal period and reports she is not passing any clots or tissue. Vital signs include a HR of 80 bpm, BP of 120/80 mm Hg, RR of 20/min, oxygen saturation of 98% on room air, and T of 98.6°F. Physical examination includes a closed cervical os with a small amount of blood in the vaginal vault. Laboratory studies include a hemoglobin of 13 g/dL, beta-human chorionic gonadotropin of 100,000 mIU/mL, and a blood type of A positive. Transvaginal ultrasound in the emergency department shows an intrauterine pregnancy with a detectable fetal heart rate and appropriate interval growth since the last ultrasound. Which of the following is the most appropriate next step in management? AAdminister progesterone BAdminister Rho(D) immunoglobulin CHospitalize for observation and bedrest DPerform serial beta-human chorionic gonadotropin measurements EProvide reassurance and outpatient follow up with ultrasound

Provide reassurance and outpatient follow up with ultrasound dx:Threatened abortion (also called threatened miscarriage) is defined by vaginal bleeding in the presence of a closed cervix without the passage of any products of conception before 20 weeks gestation The management of this presentation is expectant with reassurance and repeat outpatient ultrasound to monitor for continued viability. Patients with a threatened abortion should be educated that the pregnancy may progress normally or it could result in a miscarriage. Further, patients should be educated that most miscarriages are not preventable and that the patient is not at fault. Patients should be educated to see their obstetric clinician or return to the emergency department if they are saturating more than one pad per hour for 2 hours in a row. Some clinicians recommend pelvic rest and avoidance of strenuous physical activity until the bleeding stops. -------------------------------------------------------------- The patient does not require hospitalization for observation and bedrest (C) because there is no proven benefit and it increases the risk of developing deep vein thrombosis or pulmonary embolism.

A 33-year-old woman with no significant medical history presents to the primary care clinic for jaw weakness while chewing. She states that, for the past 6 months, she has noticed her jaw muscles feel weak after prolonged chewing. Recently, she has also noticed occasional drooping of her eyelids that varies throughout the day and can affect both eyes. The patient does not take any medications on a regular basis. Her heart rate is 82 beats/minute, blood pressure is 124/76 mm Hg, respiratory rate is 14 breaths/minute, oxygen saturation is 100%, and temperature is 97.1°F. Her physical exam reveals pupils that are equal, round, and reactive to light. You note mild ptosis of her left eyelid, and she notes subjective weakness of her jaw with repeated chewing on exam. You perform an ice pack test of her left eye, which is positive. Findings on the rest of her neurologic exam are normal. You obtain serologic testing for acetylcholine receptor antibodies and muscle-specific tyrosine kinase antibodies, which are positive. This confirms your suspected diagnosis. Which of the following is the best next step in management based on this information? ACiprofloxacin 500 mg PO BID for 7 days BIntravenous

Pyridostigmine 180 mg PO BID dx: myasthenia gravis, so pyridostigmine daily would be the most appropriate first-line therapy. She describes muscle fatigability, or muscle weakness, that worsens with prolonged muscle use

A 15-year-old girl presents for severe lower abdominal pain that occurs for the first two days of her menstrual cycle each month. She has associated nausea and diarrhea. She denies menorrhagia. She had a normal menarche at age 13 and is not sexually active. Abdominal and pelvic exams are normal. Which of the following is the initial best step in management? AObtain a Pap smear and cervical cultures BRecommend acetaminophen starting one day before menses CRecommend ibuprofen starting one day before menses DReferral for exploratory laparotomy

Recommend ibuprofen starting one day before menses dx:primary dysmenorrhea

What is the most common cause of tricuspid valve stenosis? ABacterial endocarditis BDilation and dissection of the aortic root CMarfan syndrome DRheumatic heart disease

Rheumatic heart disease Tricuspid Stenosis Most common cause: rheumatic heart disease Almost always occurs with mitral stenosis (MS) MurmurDiastolic murmur along left sternal borderLouder than MS during inspiration JVP: giant a waves -----------------------------------------------------------------

A pregnant patient with a history of bipolar disorder presents to a prenatal appointment with irritability, rapid speech, and increased energy despite insomnia for 2 weeks. Prior to the pregnancy, the patient's mood symptoms were treated effectively with valproate, risperidone, and sertraline but she is no longer taking these. Administration of which one of the following is the most appropriate next step? AAripiprazole BHaloperidol CRisperidone DSertraline EValproate

Risperidone dx;Bipolar I disorder with peripartum onset describes manic symptoms beginning either during pregnancy or within the 4 weeks following delivery. Treatment guidelines for patients with bipolar disorder who are pregnant recommend using the lowest effective doseof agents that have worked in the past, as long as those medications are relatively safe in pregnancy. Switching or trying new medications adds a risk of inefficacy, thereby exposing the fetus to potential adverse effects of both the medication and the illness itself. Since this patient has developed manic symptoms, resuming the antimanic agentrisperidone, which was previously effective, is the best next step in managing the symptoms. --------------------------------------------------------------- aripiprazole (A), a second-generation antipsychotic, might be an appropriate first-line antimanic agent during pregnancy, exposing the fetus to a medication with unknown efficacy in this patient is not recommended over starting a medication that has been previously effective. Similarly, while haloperidol (B) is well-studied in pregnancy and is an appropriate first choice for managing manic symptoms during the peripartum period, using a previously effective medication is a better option. Although sertraline (D) was part of the patient's previous medication regimen and is considered relatively safe in pregnancy, monotherapy with an antidepressant is not the recommended treatment. An antidepressant unopposed by a mood stabilizer carries a high risk of inducing or worsening manic symptoms. Although valproate (E) is an effective medication for treating acute mania, it is contraindicated in pregnancy due to the increased risk of multiple malformations and neurodevelopmental delay. Avoidance of valproate in all patients who could potentially become pregnant is often recommended as pregnancies are often unplanned.

mother brings her 9-month-old child to the clinic with a complaint of a rash on his torso for one day that began on his abdomen and spread to his arms and legs. He has had a fever up to 104°F for two days associated with a runny nose. On physical exam, his temperature is 99°F and you note a slightly erythematous papular rash on his trunk that extends to his extremities, but spares his face and palms. The patient is smiling and playing with toys in the exam room. He has associated clear rhinorrhea; patient is alert and active. Rapid antigen detection test for Group A streptococcus is negative. Which of the following is the most likely diagnosis? AErythema infectiosum BMeningococcemia CRoseola infantum DScarlet fever

Roseola infantum 6 months-3 years of age High fever lasting 3-4 days Rash that started after the fever went away PE: blanching macular or maculopapular rash with a distribution that begins at the neck and trunk region and spreads to the face and extremities Tx: supportive, self-limited

A 35-year-old woman presents to the clinic to discuss her laboratory results. The patient initially presented with amenorrhea for the past 4 months, galactorrhea, and headaches. Vital signs include a heart rate of 80 bpm, blood pressure of 120/80 mm Hg, respiratory rate of 20/minute, oxygen saturation of 98% on room air, and temperature of 98.6°F. Physical examination reveals milky nipple discharge bilaterally without palpable breast masses. Her urine pregnancy test is negative. The patient's prolactin level is 756 ng/mL. You decide to start the patient on medication to treat the suspected condition. Which of the following is the mechanism of action of the first-line medication used to treat the suspected condition? AInhibition of gonadotropin-releasing hormone BInhibition of hypothalamic dopaminergic neurons CStimulation of adrenal mineralocorticoid receptors DStimulation of gonadotropin-releasing hormone EStimulation of pituitary dopamine receptors

Stimulation of pituitary dopamine receptors dx:Pituitary adenomas . Lactotroph pituitary adenomas may initially be suspected based on symptoms of hyperprolactinemia or hypogonadism or when a pituitary adenoma is incidentally identified on magnetic resonance imaging of the brain An elevated prolactin serum concentration confirms hyperprolactinemia and is supportive of a lactotroph adenoma. Magnetic resonance imaging with contrast of the pituitary is the best imaging The initial treatment of lactotroph pituitary adenomas, including microadenomas and macroadenomas, consists of dopamine agonists, such as cabergoline or bromocriptine. These medications stimulate pituitary dopamine receptors. Cabergoline is the preferred dopamine agonist because it is more efficacious and better tolerated than bromocriptine. failed initial n consider transsphenoidal surgery, ovulation induction with clomiphene citrate (for women wishing to become pregnant), or hormonal replacement (estradiol and progesterone for women or testosterone for men).

A previously healthy 14-year-old boy presents to clinic with intermittent yellowing of the eyes. His mother is concerned because she notices this each time he has returned home from football practice this summer and wonders if he should quit the team. The boy experiences no symptoms during the episodes and was unaware of the yellowing of his eyes until his mother mentioned it. His exam is currently unremarkable. A liver panel is significant for an unconjugated bilirubin of 3 mg/dL. A CBC, peripheral smear, and reticulocyte count are unrevealing. The remainder of the panel is normal. What is the next best step in management? AObtain a hepatitis antibody panel BReferral to hematology clinic CSupportive care and reassurance DSuspension from the football team until gastroenterology referral is completed

Supportive care and reassurance dx: Gilbert syndrome. In Gilbert syndrome, the activity of uridine diphosphate glucuronosyltransferase (UGT) is decreased compared to that of the general population due to a genetic mutation. The result is a decreased rate of conjugation of serum bilirubin

A 28-year-old woman with bipolar disorder presents to the office reporting involuntary facial movements that have worsened over the past 3 months. She was prescribed risperidone and lithium about 6 months ago. On a physical exam, the patient has difficulty sticking out her tongue and has occasional involuntary movements of the tongue at rest. She also has increased blinking frequency. A blood lithium level is 0.9 mEq/L. Which of the following is the best next step in the management of her symptoms? AAdd benztropine as needed BAdd propranolol as needed CDiscontinue lithium and start valproic acid DIncrease dosage of risperidone and decrease lithium dosage ETaper and discontinue risperidone

Taper and discontinue risperidone Tardive Dyskinesia History of taking typical (first-generation) antipsychotic medication PE will show repetitive facial movements (chewing, lip smacking) Treatment is stopping the offending drug ------------------------------------------------------------------ vs Benztropine (A) is anticholinergic, which may aggravate symptoms of tardive dyskinesia.

What is the most common malignancy in men age 15-35? AHodgkin lymphoma BPapillary thyroid cancer CRenal cell cancer DTesticular cancer

Testicular cancer ---------------------------------------------------------- vs Hodgkin lymphoma (A) has a bimodal age distribution, young adulthood (age 15-35) and >55-years-old. It is less common than testicular cancer . Papillary thyroid cancer (B) has a female preponderance of 2.5:1, most commonly occurring during the fourth and fifth decades of life. Renal cell carcinoma (C) is approximately 50 percent more common in men compared with women. Renal cell carcinoma occurs predominantly in the sixth to eighth decade of life with median age at diagnosis around 64 years of age.

A 19-year-old woman presents with a recurrent outbreak of an 8 mm cluster of small vesicles at the vermillion border of her left upper lip. She reports this is the third time these lesions have developed this year and they are always preceded by a tingling sensation in the area. Which of the following is most likely to trigger a recurrence of this condition? AAlcohol use BHigh-sodium diet CPetroleum jelly DSexual intercourse EUltraviolet radiation exposure

Ultraviolet radiation exposure

Which of the following statements is most accurate regarding the history or physical exam in patients with acute ischemic heart disease? AElderly patients present more often with typical chest pain than atypical symptoms BReproducible chest wall tenderness excludes ischemia as a cause of chest pain CUp to 33% of patients diagnosed with acute myocardial infarction do not have chest pain on presentation DWomen rarely present with atypical features of acute coronary syndrome

Up to 33% of patients diagnosed with acute myocardial infarction do not have chest pain on presentation

A 55-year-old man with no history of atherosclerotic cardiovascular disease or diabetes mellitus presents to your office with questions about his cholesterol. He was previously taking atorvastatin, but stopped because his fasting low-density lipoprotein (LDL) cholesterol level dropped to 90 mg/dL and he was following the lifestyle modifications you previously recommended. Based on the 2018 American College of Cardiology/American Heart Association cholesterol management guidelines, which of the following is the most appropriate next step in his management? AEncourage the patient to continue with lifestyle modifications only BRestart atorvastatin at the previously prescribed dose CStart the patient on a different statin DUse a cardiovascular risk calculator to determine the patient's 10-year risk for atherosclerotic cardiovascular disease

Use a cardiovascular risk calculator to determine the patient's 10-year risk for atherosclerotic cardiovascular disease calculating the 10-year event risk for ASCVD as a way of initiating the conversation about prevention with patients.

A 20-year-old woman fainted while standing in line at the grocery store. The patient says she felt nauseus and diaphoretic before the episode. She reports no bowel or bladder incontinence and did not experience a postictal state. The woman in line behind her observed jerking motions of her face and fingers. She has no past medical history and does not take any medications. What is the most likely diagnosis? AHypertrophic cardiomyopathy BOrthostatic hypotension CSeizure DVasovagal syncope

Vasovagal syncope Syncope Differentiate between syncope and seizure ECG for allSan Francisco Syncope Rule (high-risk criteria): CHESS​CHFHematocrit < 30%ECG abnormalSOBSystolic BP < 90 mm Hg Adolescent athlete + syncope: HOCM (hypertrophic cardiomyopathy) Young woman + abdominal pain + syncope: ectopic pregnancy Older man + abdominal or flank pain + syncope: AAA Sudden-onset severe HA + syncope: SAH Woman + prodrome of nausea, sweating, warmth + syncope: vasovagal Malignancy + sudden-onset SOB + syncope: PE ------------------------------------------------------------- vs Seizure (C) causes a loss of consciousness, but it is associated with tonic-clonic jerking of extremities. Additionally, seizures often cause bowel or bladder incontinence as well as a post-ictal state. Syncopal events are not associated with incontinence and usually have a rapid and complete recovery.

Which of the following treatments is contraindicated in the treatment of a 5-month-old with supraventricular tachycardia? AAdenosine BIce bag to face CPropranolol DVerapamil

Verapamil Due to poor calcium reserves in the sarcoplasmic reticulum in infants, verapamil (calcium channel blocker) use in infants can cause profound hypotension and cardiovascular collapse. Therefore, it should be avoided in patients <12 months of age. Supraventricular Tachycardia (SVT) Reentrant pathway in the atrioventricular node Clinical Infants: poor feeding, fussiness, irritability Older children: palpitations, chest pain, fatigue Dx: ECG HR > 180-220 bpm without P waves (up to 300 bpm in infants), invariable Tx: vagal maneuvers, drug therapy (adenosine), and cardioversion Associated with WPW' Adenosine (A) is the first-line agent to treat supraventricular tachycardia. Applying ice to the face (B) or other vagal maneuvers can be attempted before the administration of adenosine. Propranolol (C) is a nonselective beta-blocker used in supraventricular tachycardia, but it is less effective than adenosine.

A 3-month-old infant is transferred from an outside hospital with three days of cough and congestion. Today, she developed increased work of breathing. Exam was significant for intercostal retractions, diffuse soft expiratory wheezing, and coarse crackles. A chest radiograph shows bilateral hyperinflation and areas of atelectasis with volume loss. What is the most likely diagnosis? ABacterial pneumonia BStatus asthmaticus CViral bronchiolitis DViral pneumonia

Viral bronchiolitis 1-3 day prodromal URI symptoms PE: tachypnea, retractions, polyphonic wheezing, and rales Diagnosis is made by history and physical exam Most commonly cause: respiratory syncytial virus (RSV) Treatment is supportive care

A previously healthy 22-year-old woman presents to your office with a recurrent, painful genital rash over the past 4 months. She has no significant past medical history, is sexually active, and does not take any medications. Physical examination of the genitalia reveals grouped, erythematous, tender vesicles on the patient's labia majora. The lesions include some intact and some ruptured vesicles with ulcerations. Which of the following diagnostic studies is most appropriate? AClinical diagnosis based on appearance BSerum antibody test CSkin biopsy DTzanck smear EViral culture

Viral culture herpes simplex virus --------------------------------------------------------- A Tzanck smear (D) cannot distinguish herpes simplex virus from varicella-zoster virus and is only helpful if positive. It is not the test of choice for diagnosing genital herpes.

A man presents with difficulty walking and painful calves. He has a history of alcohol use disorder and ulcerative colitis, both untreated. He reports no history of chronic diarrhea. A blood tox screen is negative for alcohol and illicit drugs. His examination is significant for several neurologic findings: altered proprioception, distal numbness, hyporeflexia, confusion, and nystagmus. Perioral and tongue examination is normal. These symptoms are most likely related to a deficiency of which of the following vitamins? AB1 BB2 CB3 DC

Vitamin B1, thiamine, is necessary for energy, carbohydrate, lipid, and amino acid metabolism. It acts as a component of the coenzyme responsible for conversion of pyruvate to acetyl-CoA. Thiamine is also essential for proper nerve and muscle function.

A 19-year-old man is brought to the ED via ambulance after he was found sleeping outside in a snowstorm. The ambient temperature is 25°F (-3.9°C). His vital signs are T 36.0°C, BP 125/70 mm Hg, RR 14/min, and HR 75 bpm. He reports stinging and burning pain in all ten fingers. On exam, you note edema, erythema, and multiple blisters beginning to form over the fingers and hands. Which of the following is the most appropriate next step in management? AAdminister intravenous antibiotics BDebridement of blisters CDry rewarming at 98.6°F (37°C) DWet rewarming at 98.6°F (37°C)

Wet rewarming at 98.6°F (37°C) dx: (second-degree) frostbite. Rapid rewarming is the cornerstone of frostbite therapy and should be initiated as soon as possible. The injured extremity should be placed in gently circulating water (ideally a whirlpool) at a temperature of 98.6-102.2°F (37-39°C) for approximately 15-30 minutesuntil the distal extremity is pliable and erythematous.

The ECG is a reliable diagnostic tool Pericarditis Sx: pleuritic chest pain radiating to the back that is worse when lying back and improved when leaning forward PE: tachycardia and pericardial friction rub, distant heart sounds ECG: PR depression, PR elevation (aVR), diffuse ST segment elevation (concave) Causes: idiopathic, viral Tx: NSAIDs, colchicine

Which of the following statements is true regarding the condition associated with the ECG seen above? APain is often relieved in the supine position BThe ECG is a reliable diagnostic tool CThrombolytic therapy is a potential treatment DVentricular dysrhythmias are common

A 10-year-old boy collapses on the football field and is quickly evaluated by the sports medicine doctor. He had been practicing on the field for approximately 2 hours without any rest. His temperature is 105°F and his skin is sweaty. On testing his mental status, he can say his name and age but does not know where he is. Which of the following is the best initial treatment for this patient? ACool the body with fans BOffer oral rehydration solution CProvide cold water to drink DWhole body cold-water immersion

Whole body cold-water immersion dx: Heat Stroke Loss of thermoregulatory mechanisms Dry skin in older patients, moist when due to physical activity Altered mental status Abnormal LFTs Rapid cooling to 39°C: evaporative or cold water immersion Avoid antipyretics . Treatment of children with heat stroke consists of immediate whole body cooling, usually performed by cold water immersion. Rapid cooling should be continued until the temperature decrease to 101 or 102°F. Careful monitoring of the airway, breathing, circulation, temperature, and CNS status is crucial. IV fluids should also be initiated as soon as possible, at a rate of 800 mL/m2 in the first hour. In contrast, classic heat stroke is seen in elderly patients and is usually of a slower onset, with the physical exam notable for dry, hot skin ------------------------------------------------------------- vs Heat exhaustion is a moderate heat illness characterized by a core body temperature of 100-103°F. CNS dysfunction is usually not present, or may be mild. Patients may exhibit headache, gastrointestinal symptoms such as nausea or vomiting, dizziness, weakness, piloerection, orthostasis, and possibly syncope. In this case, play should be stopped and the patient should be moved to a cool environment. The body can be cooled with fans (A), and ice should be placed over the groin or axillae. Oral rehydration (B) should be offered in the setting of heat exhaustion, but if not tolerated, IV fluids should be initiated. If the patient develops signs of heat stroke, he should be transported immediately to an emergency facility. Heat cramps are the most common heat injury and usually occur in the setting of mild dehydration and salt loss. The calf and hamstring muscles are often affected. Treatment consists of oral rehydration with electrolyte solutions and gentle stretching. Cold water (C) should be offered to all children participating in physical activity in the heat. This is appropriate for all children exercising for less than 1 hour. However, fluids with electrolytes and sugar should be offered for those exercising over 1 hour, as drinking only water may put them at risk for hyponatremia.

Adenocarcinoma Bowel adhesions following abdominal surgery are the most common cause of small bowel obstruction. What is the string of pearls sign? Answer: Small round pockets of air that line up (look like a string of pearls) on the abdominal plain film in the setting of a small-bowel obstruction.

n the United States, which of the following conditions, other than postoperative adhesions, is most likely responsible for causing the diagnosis seen in the image above? AAdenocarcinoma BCrohn disease CIntussusception DVolvulus


Related study sets

RE lecture 2 strengthening/resistance exercises

View Set

Chapter 7 Monopoly Practice Quiz

View Set

PSY 101 - Chapter 10 - Personality: Theory and Measurement

View Set

US History Chapter 2,3,4 Study Guide

View Set

Digital Information Technology Final Exam Flash Cards Olivia Freiberg

View Set

Physics 110- Pre-Lecture Assignment 1

View Set

Cognitive Psych Quiz One: Chapter 1

View Set